Download as pdf or txt
Download as pdf or txt
You are on page 1of 111

Bachelor of Science in Mathematics

DIFFERENTIAL EQUATION
[BMSS-32]
SEMESTER - III

Department of Mathematics
School of Sciences
Tamil Nadu Open University
577, Anna Salai, Saidapet,
Chennai – 600 015, Tamil Nadu.
Name of the Programme: B.Sc., Mathematics
Course Code: BMSS-32
Course Title: Differential Equations

Curriculum Design
Dr. E. Kumar
Assistant Professor
School of Sciences
Tamil Nadu Open University, Chennai – 15

Course Writer
Dr. E.J. Lalith Kumar,
Head of Department
Department of Mathematics
SRM Arts & Science College
Kattankulathur,
Chennai – 603 203.

Course Coordinator
Dr. E. Kumar
Assistant Professor
School of Sciences
Tamil Nadu Open University, Chennai – 15

Total No. of Pages: 113


First Edition: December, 2022

ISBN No: 978-93-5706-421-7

©Tamil Nadu Open University


All rights reserved. No part of this work can be reproduced in any form, by mimeograph or any other
means, without permission in writing from the Tamil Nadu Open University. Course Writer is the
sole responsible person for the contents presented/available in the Course Materials. Further
information on the Tamil Nadu Open University Academic Programmes may be obtained from the
University Office at 577, Anna Salai, Saidapet, Chennai-600 015 / www.tnou.ac.in

@ TNOU, 2021, “Differential Equations” is made available under a Creative


Commons Attribution -Share Alike 4.0 License (International)
http://creativecommons.org/licenses/by-sa/4.0
Printed by:
Tamil Nadu Open University
Department of Mathematics
School of Science,
Chennai – 15

B.Sc., Mathematics - Syllabus – II year – III Semester (Distance Mode)


COURSE TITLE : DIFFERENTIAL EQUATIONS
COURSE CODE : BMSS- 32

COURSE CREDIT : 4

COURSE OBJECTIVES

While studying the DIFFERENTIAL EQUATIONS, the Learner shall be able to:
 To evaluate integration of irrational functions and improper integrals
 To understand the concepts of double and triple integration

COURSE OUTCOMES
After completion of the DIFFERENTIAL EQUATIONS, the Learner will be able to:
 understand the concepts of double and triple integration
 use Beta-Gamma functions as a tool to evaluate integrals
 use numerical integration for approximating the integrals that are
difficult or impossible to integrate analytically

Block I

Differential equation of first order – formation – variable separable – Homogeneous


– Linear – Bernouille – Equations solvable for x,y,p – Clairaut’s form – Exact
equation of first order.

Block II

Differential equation of second order – Differential equation with constant


coefficients – various types of particular integral.
Block III

Linear Differential equation with variable coefficients – equations reducible to linear


homogeneous equations – variation of parameters. Method of undetermined co-
efficient,

Block IV

Simultaneous differential equation of the form dx/P = dy/Q = dz/R – Exactness, nth
order exact differential equation – Condition of exactness for a nth order linear
equation.

Block V

Partial differential equations of the first order – classification of integrals –


derivations of Partial differential equation – Lagrange’s method of solving –
Charpit’s method - the linear equations – standard forms – Equations reducible to
the standard forms.

Reference Books

1. S. Narayanan and T.K. Manickavasagam Pillai, Differential Equations & its


Applications, S. Viswanathan Publishers Pvt Ltd, 2003.

2. P. Kandasamy and K. Thilagavathi, Mathematics for B.Sc., Volume III, S.


Chand & Co, New Delhi, 2004.

3. S.Arumugam and A.Issac, Differential Equations with Applications, New


Gamma Publishing House, 2002.

4. S.Narayanan & T.K.Manicavasagam pillai, Calculus Vol- III, S.Viswanathan


Pvt Ltd, 1991.
Contents
S.No. Title Page
No
1 BLOCK – I
1.1 Unit – 1 Equations of the First Order and First Degree 1
1.2 Unit – 2 Formation of Differential Equations 25
1.3 Unit – 3 Equations of the First order and Higher Degree 28
2 BLOCK - II
2.1 Unit – 4 Linear Equations with Constant Coefficients 36
2.2 Unit – 5 Various Types of Particular Integral 40
3 BLOCK – III
3.1 Unit – 6 Linear Differential Equation With Variable Coefficients 56
3.2 Unit – 7 Equations Reducible to the Linear Equations 64
3.3 Unit – 8 Variation of Parameters 68
3.4 Unit – 9 Method of Undetermined Coefficient 71
4 BLOCK - IV
4.1 Unit – 10 Simultaneous Differential Equation of the Form 73
4.2 Unit – 11 nth Order Exact Differential Equations 79
5 BLOCK – V
5.1 Unit – 12 Partial Differential Equations of the First Order 83
5.2 Unit – 13 Derivation of Partial Differential Equation 85
5.3 Unit – 14 Lagrange’s Method of Solving the Linear Equation 89
5.4 Unit – 15 Charpit’s Method 94
5.5 Unit – 16 Standard Forms 97
5.6 Unit – 17 Equations reducible to the Standard Form 102
Appendix – Plagiarism Certificate
1

BLOCK- I
UNIT 1
EQUATIONS OF THE FIRST ORDER AND FIRST DEGREE

Objectives
After successful completion of this unit, students will be able to solve the differential
equations of first order and of first degree using the following types

 Variable Separable
 Homogeneous Equations
 Non-Homogeneous Equations
 Linear Equations
 Bernouilli’s Equation
 Exact Differential Equations

1.1.1 TYPE A : Variable Separable

dy
1. Solve  xy  x  y  1
dx

dy
Solution: Given  xy  x  y  1
dx

 x( y  1)  ( y  1)
 ( y  1)( x  1)
dy
  ( y  1)( x  1)
dx
dy
  ( x  1)dx
( y  1)

On integrating both sides, we get

dy
 ( y  1)   ( x  1)dx
x2
log( y  1)   xc
2

which is the required solution


2

3
2. Solve ydx  xdy  3x y e dx  0
2 2 x

Solution: Given that ydx  xdy  3x y e dx  0


2 2 x

 ydx  xdy  3 x 2 y 2 e x dx
ydx  xdy 3

   3 x 2 2 x
y e dx
y2
On integrating, we get

ydx  xdy 3

 y 2
   3x 2 e x dx

x
  d      eu dx  If u  x ,
3
du  3x 2 dx 
 y

x
  e u  c
y

x x3
 e  c
y

which is the required solution


1/2
dy  1  y 2 
3. Solve   0
dx  1  x 2 

1/2
dy  1  y 2 
Solution: Given   0
dx  1  x 2 

1/2
dy  1 y2 
  2 
dx  1 x 
dy 1 y2

dx 1  x2
dy dx

1 y2 1  x2

on integrating we get
dy dx
 1  y 2    1  x2
sin 1 y   sin 1 x  c
sin 1 y  sin 1 x  c

which is the required solution


3

dy
4. Solve x 1  y 2  y 1  x 2 0
dx

dy
Solution: Given that x 1  y 2  y 1  x 2 0
dx

dy
 y 1  x2   x 1 y2
dx

 y 1  x 2 dy   x 1  y 2 dx

y dy x dx
 
1 y 2
1  x2

  y 1  y 2  dy    x 1  x 2 
1/2 1/2
dx

  d ( 1  y2 )    d ( 1  x2 )

 1  y 2   1  x2  c

 1  y 2  1  x2  c

which is the required solution

Check Your Progress:

Solve the following equations:

1. e x tan ydx  (1  e x )sec2 ydy  0


2. 1  x 2 dx  1  y 2 dy  0
3. sec2 x tan ydx  sec2 y tan xdy  0
4. ( x 2  yx 2 )dy  ( y 2  xy 2 )dx  0

Answers:

1. tan y  c(1  e x )


2. x 1  x 2  y 1  y 2  log x  1  x 2  y  
1 y2  c
3. tan x tan y  c
 x 1 1
4. log        c
 y x y
4

1.1.2 TYPE B : Homogeneous Equations.

5. Solve xdy  ydx  x 2  y 2 dx

Solution: Given xdy  ydx  x 2  y 2 dx

 xdy  ydx  x 2  y 2 dx
xdy  ( y  x 2  y 2 )dx
dy ( y  x 2  y 2 )
  (1)
dx x

The above equation is of homogeneous equation.

Put y  vx

dy dv
 v x
dx dx
so equation (1) becomes
dv vx  x 2  v 2 x 2
v x 
dx x
dv vx  x 2 (1  v 2 )
v x 
dx x

dv vx  x (1  v 2 )
v x 
dx x

dv x v  (1  v )
v x 
2

dx x
dv
v  x  v  (1  v 2 )
dx
dv
x  (1  v 2 )
dx

dv dx
 (1  v 2 )

x

log(v  (1  v 2 ))  log x  log c

log(v  (1  v 2 ))  log xc

v  (1  v 2 )  xc
2
y  y
 1     xc
x x
5

y x2  y 2
  xc
x x

y  x2  y 2
 xc
x

y  x 2  y 2  cx 2

which is the required solution

dy
6. Solve ( x  y )  xy
2 2

dx

dy
Solution: Given that ( x  y )  xy
2 2

dx

dy xy
 2  (1)
dx x  y 2

This is of homogeneous type

Put y  vx
dy dv
 v x
dx dx
so equation (1) becomes
dv x(vx)
v x  2 2 2
dx x  v x
dv x 2v
v x  2
dx x (1  v 2 )
dv v
v x 
dx 1  v 2

dv v
x  v
dx 1  v 2
dv v  v  v 3
x 
dx 1 v2
dv v3
x 
dx 1 v2
1  v 2 dv dx
3

v x

1 1 dx
v 3
dv   dv   
v x
6

1
  log v   log x  c
2v 2
x2 y
 2  log   log x  c
2y x
x2
  log y  log x   log x  c
2 y2
x2
  log y  c
2 y2

which is the required solution

dy dy
7. Solve y  x  xy
2 2

dx dx

dy dy
Solution: Given that y 2  x2  xy
dx dx

dy dy
y 2  xy  x2
dx dx
dy
y2  ( xy  x 2 ) (or )
dx
dy y2
  (1)
dx xy  x 2

This is of homogeneous equation

Put y  vx
dy dv
 v x
dx dx
so equation (1) becomes
dv v2 x2
v x 
dx x(vx)  x 2
dv x 2v 2
v x  2
dx x (v  1)
dv v 2
v x 
dx v  1

dv v 2
x  v
dx v  1

dv v 2  v 2  v
x 
dx v 1
7

dv v
x 
dx v  1
v 1 dx
dv 
v x
1 1
 dv   v dv   x dx
v  log v  log x  log c
y y
 log  log x  log c
x x
y
 log y  log x  log x  log c
x
y
 log c  log y
x

y y
 log cy (or ) log cy 
x x
 cy  e y/ x

1
 y  e y/ x
c

which is the required equation

dy x  y
8. Solve 
dx x  y

dy x  y
Solution: Given that   (1)
dx x  y

This is of homogeneous equation

Put y  vx
dy dv
 v x
dx dx
so equation (1) becomes
dv x  vx
v x 
dx x  vx
dv x(1  v)
v x 
dx x(1  v)

dv 1  v
v x 
dx 1  v
dv 1  v
x  v
dx 1  v
8

dv 1  v  v  v 2
x 
dx 1 v
dv (v  2v  1)
2
x 
dx 1 v
(1  v) dx
dv 
v  2v  1
2
x
1 2v  2 1

2 v  2v  1
2
dv    dx
x
2v  2 1
 v 2  2v  1 dv   2 x dx
log(v 2  2v  1)   2 log x  log c
log(v 2  2v  1)   log x 2  log c
log(v 2  2v  1)  log x 2  log c
 (v 2  2v  1) x 2  c
y2 y
 ( 2  2  1) x 2  c
x x
( y  2 xy  x 2 ) x 2
2
 c
x2
 y 2  2 xy  x 2  c

which is the required solution

Check your Progress:

Solve the following equations:

1. ( x  y ) 2 dx  2 x 2 dy
2. ( x3  y 3 )dx  3 xy 2 dy  0
dy
3. x( y  x)  ( x  y) y
dx
dy
4. x  y  x 2  y 2
dx
Answers:

 y
1. log x  2 tan 1    c
x
2. x3  2 y 3  cx
y
3. e  cxy
x

4. y  x 2  y 2  cx 2
9

1.1.3 TYPE C : Non-Homogeneous Equations of the first degree in x and y

dy x  2 y  3
9. Solve 
dx 2 x  y  3

dy x  y
Solution: Given that   (1)
dx x  y

Here a  1, b  2; a '  2, b ' 1


a b
(ie., ) 
a' b'
 put x  X  h and y  Y  k
dx  dX dy  dY

dY ( X  h)  2(Y  k )  3
Hence (1)  
dY (2 X  h)  2(Y  k )  3
dY ( X  2Y )  (h  2k  3)
  (2)
dY (2 X  Y )  (2h  k  3)

Now we can choose h and k such that h  2k  3  0 and

2h  k  3  0
By solving the above equations using cross multiplication method, we get

2 3 1 2
1 3 2 1
h k 1
(ie., )  
6  3 6  3 1  4
h k 1
  
3 3 3
 h  1 and k  1
dY X  2Y
Therefore equation (2) becomes   (3)
dX 2 X  Y
This is homogeneous.

so put Y  vX
dY dV
 v X
dX dX

Therefore equation (3) becomes


dV X  2vX X (1  2v)
v X  
dX 2 X  vX X (2  v)
dV 1  2v 1  2v  2v  v 2
X  v
dX 2v 2v
10

dV 1  v 2
X 
dX 2v
2v 1
 dv  dX  (4)
1 v 2
X

2v 2v A B 1/ 2 3 / 2 1  1 3 
Here         
1 v 2
(1  v)(1  v) 1  v 1  v 1  v 1  v 2  1  v 1  v 

1  1 3  1
Equation (4) becomes    dv   dx
2  1 v 1 v  X
1
log(1  v)  3log(1  v)  log X  log c
2
1
log(1  v)  log(1  v)3   log X  log c
2
1 v
log  2 log cX
(1  v)3
1 v
 log  cX 
2
log
(1  v) 3

1 v
 X 2 c1
(1  v) 3

Y X Y
1
X  X 2c  X  X 2 c1
 X Y 
1 3
Y 3
(1  )
X X3
on simplification, we get
X Y
 c1
 X Y 
3

 X  Y  c1  X  Y 
3

 ( x  h  y  k )  c1 ( x  h  y  k )3
 x  y  2  c1 ( x  y )3 ( h  1 and k  1)

10. Solve ( x  y  1)dy  ( x  y  1)dx

dy ( x  y  1)
Solution: Given that   (1)
dx ( x  y  1)

Here a  1, b  1; a '  1, b '  1


a b
(ie., ) 
a' b'
 put x  y  z (or )
zx y
dz dy dy dz
 1  (or )  1
dx dx dx dx
11

Therefore equation (1) becomes

dz z 1
1 
dx z 1
dz z  1 z 1  z 1 2z
  1 
dx z  1 z 1 z 1
z 1
dz  dx
2z
z 1
dz  2dx
z
 1
 1  z  dz  2 dx

z  log z  2 x  c
( x  y )  log( x  y )  2 x  c
 ( x  y )  log( x  y )  c1
which is the required solution.
Check your Progress:

Solve the following equations:

dy
1. (2 x  4 y  3)  ( x  2 y  1)  0
dx
dy x  7 y  2
2. 
dx 3 x  5 y  6
3.( x  y )dy  ( x  y  1)dx

Answers:

1. log{4( x  2 y )  5}  4( x  2 y )  c1
( y  x  2) 4
2. log  3( x  2)  c
( x  2)3 ( x  5 y  2)
2 y 1 1 1
3.tan 1  log( x 2  y 2  x  y  )  c
2x 1 2 2
12

1.1.4 TYPE D : Linear Equation

A differential equation is said to be linear when the dependent variable and its derivatives
occur only in the first degree.

dy
General form:  Py  Q  (1)
dx
( where P and Q are functions of x only )
Its Solution is :
y( I .F )   Q ( I .F ) dx  c where I is the Integrating Factor whichis equal toe
Pdx

dy 1
11. Solve  y cos x  sin 2 x
dx 2

dy 1
Solution: Given that  y cos x  sin 2 x  (1)
dx 2

dy
Equation (1) is of the form  Py  Q
dx

1
Here P  cos x Q  sin 2 x
2

The general solution is ye   Qe


Pdx Pdx
dx  c

 Pdx   cos xdx  sin x


 e
Pdx
 esin x

1
Hence the solution is yesin x   sin 2 xesin x dx  c  (2)
2

1
Since sin 2 x  2sin x cos x, sin 2 x  sin x cos x
2

Equation (2) becomes yesin x   sin x cos xesin x dx  c

If we substitute z  sin x on RHS , dz  cos xdx

 yesin x   ze z dz  c
 ze z  e z  c
 e z ( z  1)  c
 yesin x  esin x (sin x  1)  c

which is the required solution.


13

dy
12. Solve (1  x )  2 xy  x 1  x 2 given that y = 0 when x = 0.
2

dx

dy
Solution: Given that (1  x )  2 xy  x 1  x 2
2

dx

Dividing the given by (1  x 2 ) on both sides, we get

dy 2 xy x 1  x2
 
dx (1  x 2 ) (1  x 2 )
dy 2 xy x
    (1)
dx (1  x )
2
1  x2

dy 2x x
The above equation is of the form  Py  Q where P  ; Q
dx (1  x )
2
1  x2

2x 2 x
  Pdx   dx    dx   log(1  x 2 )  log(1  x 2 ) 1
(1  x )
2
(1  x )
2

e
Pdx 2 1 1
 elog(1 x )
 (1  x 2 ) 1 
1  x2

Therefore the solution is ye   Qe


Pdx Pdx
dx  c

1 x 1
y  dx  c
1 x 2
1  x2 1  x
2

y x
 dx  c
1 x 2
(1  x 2 )3/2

By substituting x  sin  , dx  cos  d on RHS

y sin  cos  d
 c
1 x 2
(1  sin 2  )3/2
sin  cos  d
 c
(cos 2  )3/2
sin  cos  d
 c
cos3 
sin  1
 d  c
cos  cos 

  tan  sec  d  c
 sec  c
1
  c (since x  sin  , 1  x 2  cos  )
1 x 2
14

y 1
(ie)   c  (2)
1 x 2
1  x2

Given that when x = 0, y = 0

Therefore equation (2) becomes

0 1
 c
1 0 2
1  02
 0 1  c
 c  1

y 1
(2)    1 which is the required solution.
1 x 2
1  x2

Check your Progress:

Solve the following equations:

dy
1.  y tan x  cos3 x
dx
1
dy 1 log x
2. x  y log x  e x 2x

dx
dy
3. (1  x 2 )  2 xy  cos x
dx
Answers:
1. 4 y  cos x(2 x  sin 2 x  c)
2. e x  c
3. y (1  x 2 )  sin x  c

1.1.5 TYPE E : Bernoulli’s Equation

dy
The given equation will be  Py  Q y n  (1)
dx

( where P and Q are functions of x only )

This equation can be reduced to linear form (Type D), dividing by yn on both sides.

dy
yn  Py1 n  Q
dx

By substituting
dz
z  y  n the above equation becomes  Pz (1  n)  Q (1  n) (This is linear in z )
dx
15

dy y
13. Solve   y2
dx x

dy 1
Solution: Given that  y  y2
dx x

dy 1
Dividing both sides by y 2 , we get y 2  y 1 1  (1)
dx x

dz dy
Now we can substitute y 1  z , so z  y 1 and   y 2
dx dx

dz 1 dz 1
 (1)    z  1 (or )  z  1 ( which is linear in z )
dx x dx x
1
  dx
1
Here P   ; Q   1, so I .F  e  Pdx 1
 e x  e log x  elog x  x 1 
1
x x

Therefore the solution is Z ( I .F )   Q ( I .F ) dx  c

1 1
z     (1)   dx  c
 x  x
1 1
y 1      dx  c
x x
1
  log x  c
xy

1
 log x  c
xy

which is the required solution.

dy sin x cos 2 x
14. Solve  y tan x 
dx y2

dy sin x cos 2 x
Solution: Given that  y tan x 
dx y2

dy
y2  y 3 tan x  sin x cos 2 x  (1)
dx
Put z  y 3
dz dy dy 1 dz
 3y2  y2 
dx dx dx 3 dx
1 dz
so (1) becomes  z tan x  sin x cos 2 x
3 dx
16

dz
 3z tan x  3sin x cos 2 x
dx
Here P   3tan x ; Q  3sin x cos 2 x

 sin x
I.F = e  e  e  cos x  e3log(cos x )  elog(cos x )  cos3 x
Pdx 3tan xdx 3 dx 3

Therefore the solution is Z ( I .F )   Q ( I .F ) dx  c

z (cos3 x )   3sin x cos 2 x ( cos3 x ) dx  c


  3sin x cos5 x dx  c
 3 cos5 x d (cos x)  c
cos 6 x
 3 c
6
cos6 x
z cos3 x   c
2
cos3 x
y 
3
c
2
2 y 3   cos3 x  c
2 y 3  cos3 x  c

which is the required solution.

Check your Progress:

Solve the following equations:

dy
1. (1  x)  1  2e  y
dx
dy
2.  2 y tan x  y 2
dx
dy
3.  x 2 y 2  xy
dx

Answers:

1. e y (1  x)  2 x  c
1 sin 2 x 
2. cos 2 x   x   y  cy
2 2 
3. y 2  x 2  1  ce x
2
17

1.1.6 TYPE F : Exact Differential Equations.

M N
The equation Mdx  Ndy  0 is exact if 
y x

Here the solution is 


y ( const )
Mdx   (terms in N not containing x)dy  c

15. Solve (a 2  2 xy  y 2 )dx  ( x  y)2 dy  0

Solution: Given (a 2  2 xy  y 2 )dx  ( x  y)2 dy  0

The given equation is of the form Mdx  Ndy  0

M
Here M  a 2  2 xy  y 2 ,   2 x  2 y   2( x  y )
y

N
N  ( x  y ) 2 ,   2( x  y )
x

M N
(i.e) 
y x

Therefore the given equation is exact

The solution is 
y ( const )
Mdx   (terms in N not containing x)dy  c


y ( const )
(a 2  2 xy  y 2 )dx    y 2 dy  c


y ( const )
(a 2  2 xy  y 2 )dx    y 2 dy  c

x2 y3
a x  2y  y x   c
2 2

2 3
3
y
a 2 x  x 2 y  xy 2   c
3

which is the required solution

16.Solve
(2 x 2 y  4 x3  12 xy 2  3 y 2  xe y  e2 x )dy  (12 x 2 y  2 xy 2  4 x3  4 y 3  2 ye2 x  e y )dx  0

Solution: The given equation is of the form Mdx  Ndy  0

Here M  12 x 2 y  2 xy 2  4 x3  4 y 3  2 ye2 x  e y

N  2 x 2 y  4 x3  12 xy 2  3 y 2  xe y  e2 x
18

M
  12 x 2  4 xy  12 y 2  2e 2 x  e y
y
N
 4 xy  12 x 2  12 y 2  e y  2e 2 x
x
M N
(ie) 
y x

Therefore the given equation is Exact.

The solution is 
y ( const )
Mdx   (terms in N not containing x)dy  c


y ( const )
(12 x 2 y  2 xy 2  4 x3  4 y 3  2 ye 2 x  e y )dx   3 y 2 dy  c

12 x3 y x2 x4 y3
  2 y 2  4  4 y 3 x  ye2 x  xe y  3  c
3 2 4 3
 4 x y  x y  x  4 xy  e y  xe  y  c
3 2 2 4 3 2x y 3

which is the required solution

Integrating Factor:

It the given equation is not exact, then we can multiply by some suitable function of x and y
to make exact. Such a function is called an “Integrating Factor”.

Example: If the given equation is ydx  xdy  0  (1)

My N  x
M N
1  1
y x
M N
(ie) 
y x

Therefore the given equation is not exact.

NOTE:
1 ydx  xdy x
1. If we multiply equation (1) by 2
, we get 2
 0  d    0 which is exact.
y y  y
1 ydx  xdy  y
2. If we multiply equation (1) by 2 , we get  0  d    0 which is exact.
x
2
x x
1 ydx  xdy
3. If we multiply equation (1) by , we get  0  logx  log y  0 whichis exact.
xy xy
1 1 1
So 2
, 2 and are the integrating factors of equation (1)
y x xy
19

Rules for finding Integrating Factors:

Sl.No Conditions Integrating Factor


M N 1 1 1
1 If  or 2 or
y x y 2
x xy
If the given equation is not exact and it is Homogeneous 1
2 (provided Mx  Ny  0 ) Mx  Ny
If the given equation is not exact and it is of the form 1
3
f1 ( xy) y dx  f 2 ( xy) x dy  0 (provided Mx  Ny  0 ) Mx  Ny
1  M N 
If    is a function of x alone e
f ( x ) dx
4
N  y x 
1  N M 
If    is a function of y alone e
f ( y ) dy
5
M  x y 

17. Solve a( xdy  2 ydx)  xydy

Solution: Given that ax dy  2ay dx  xydy  0  (1)

2ay dx  (ax  xy )dy  0


Here M  2ay N  ax  xy
M N
 2a a y
y x
M N
(ie) 
y x

(i.e.,) The given equation is not exact.

If we multiply the given equation by 1/xy, we get

ax dy  2ay dx  xydy
0
xy
1 1
a dy  2a dx  dy  0
y x
1 1
a  dy  2a  dx   dy  0
y x

a log y  2a log x  y  c
a(log y  2 log x)  y  c
a(log y  log x 2 )  y  c
a(log yx 2 )  y  c

which is the required solution


20

18. Solve ( x 2 y  2 xy 2 )dx  ( x3  3x 2 y)dy  0

Solution: By comparing the given equation with Mdx  Ndy  0 we get

M  x 2 y  2 xy 2 N   x3  3x 2 y
M N
 x 2  4 xy  3 x 2  6 xy
y x
M N
(ie) 
y x

(ie)., The given equation is not Exact. Since the given equation is homogeneous, the
Integrating

1
Factor is Mx  Ny

1 1 1
So  2  2 2
Mx  Ny ( x y  2 xy ) x  (3x y  x ) y x y
2 2 3

1
By using the integrating factor in the given equation, we get
2
x y2

1 1
2 2
( x 2 y  2 xy 2 )dx  2 2 ( x 3  3x 2 y )dy  0
x y x y
 1 2  x 3
   dx   2   dy  0
 y x y y

This equation becomes exact and which is of the form Mdx  Ndy  0 . Therefore the solution
is 
y ( const )
Mdx   (terms in N not containing x)dy  c

1 2 3
    dx   dy  c
y ( const ) 
y x y
x
 2log x  3log y  c
y

x
 log x 2  log y 3  c
y
x y3
 log 2  c
y x

which is the required solution.


21

19. Solve y( xy  2 x 2 y 2 )dx  x( xy  x 2 y 2 )dy  0

Solution: Given that y( xy  2 x 2 y 2 )dx  x( xy  x 2 y 2 )dy  0


By comparing the given equation with Mdx  Ndy  0 we get

M  xy 2  2 x 2 y 3 N  x 2 y  x3 y 2
M N
 2 xy  6 x 2 y 2  2 xy  3 x 2 y 2
y x
M N
(ie) 
y x
(ie)., The given equation is not Exact. But the given equation is of the form

1
f1 ( xy) y dx  f 2 ( xy) x dy  0 . So the integrating factor is
Mx  Ny

1 1 1
I.F =  2 2  3 3
Mx  Ny x y  2 x y  x y  x y 3x y
3 3 2 2 3 3

1
So multiply the I.F in the given equation, we get
3x3 y 3
1
( xy 2  2 x 2 y 3 )dx  ( x 2 y  x3 y 2 )dy   0
3 3 
3x y
 1 2   1 1 
  2   dx   2
  dx  0
 3x y 3x   3xy 3 y 

This equation becomes exact and which is of the form Mdx  Ndy  0 . Therefore the solution
is 
y ( const )
Mdx   (terms in N not containing x)dy  c

 1 2  1
  2   dx   ( )dy  c
y ( const ) 
3x y 3x  3y
1 2 1
  log x  log y  c
3 xy 3 3
1 1 
    2 log x  log y   c
3  xy 
1 1 
    log x 2  log y   c
3  xy 
1 x2
   log  3c  c1
xy y
x2 1
 log   c1
y xy

which is the required solution.


22

20. Solve (1  xy 2 )dx  (1  x 2 y)dy  0

Solution: The given equation is of the form Mdx  Ndy  0

M 1  xy 2 N  1  x2 y
M N
 2 xy  2 xy
y x
M N
(ie) 
y x

The given equation is exact.

Therefore the solution is 


y ( const )
Mdx   (terms in N not containing x)dy  c


y ( const )
(1  xy 2 )dx   dy  c

x2 y 2
x yc
2
1
( xy ) 2  x  y  c
2

which is the required solution.

21. Solve ( x 2  y 2  2 x)dx  2 ydy  0

Solution: The given equation is of the form Mdx  Ndy  0

M  x2  y 2  2x N  2y
M N
2y 0
y x
M N
(ie) 
y x

M N
The given equation is not exact, but   2y
y x

1  M N  1
    2 y 1 (it is a fucntion of x)
N  y x  2 y

 I.F = e  e  e x
f ( x ) dx 1dx

By multiplying the I.F e x in the given equation, we get


23

e x ( x 2  y 2  2 x)dx  2e x ydy  0
M N
 2e x y  2e x y
y x
M N
 ( It is exact )
y x
Therefore the solution is 
y ( const )
Mdx   (terms in N not containing x)dy  c


y ( const )
e x ( x 2  y 2  2 x)dx   (0)dy  c

  x 2 e x dx   e x y 2 dx  2 xe x dx  c

 ( x 2e x )  (2 xe x )  (2e x )   e x y 2  2 ( xe x )  (e x )   c


 ex ( x2  y 2 )  c

Check Your Progress:

Solve the following equations:

1. ( x 2  y 2 )( xdx  ydy )  a 2 ( xdy  ydx)


3
2. ydx  xdy  3x 2 y 2e x dx  0
3. ( x 2  x  y 2 )dx  ( ye y  2 xy )dy  0
4. ( y  3 x 2 ) dx  x(1  xy 2 )dy  0

Answers:

1 2  y
1. ( x  y 2 )  a 2 tan 1    c
2 x
3
2. ye x  x  c
x3 x 2
3.   y 2 x  e y ( y  1)  c
3 2
y 1
4.   3 x  y 3  c
x 3
24

UNIT- 2

FORMATION OF DIFFERENTIAL EQUATIONS

Objective
 After successful completion of this unit, students will be able to form the
differential equation by eliminating the arbitrary constants from the given equation

1.2.1 Method to form the differential equation from the given equation by eliminating
arbitrary constants.

(i) Write down the given equation.

(ii) Differentiate the given equation w.r.t ‘x’ as many times as the number of arbitrary
constants

(iii) Form the equation by eliminating the arbitrary constants.

1. Form the differential equation by eliminating  and  from ( x   )2  ( y   )2  r 2

Solution: Given that ( x   )2  ( y   )2  r 2  (1)

Differentiating (1) w.r.t ‘x’ twice, we get

dy
2( x   )  2( y   ) 0
dx
dy
(x  )  ( y   ) 0  (2)
dx

Differentiating again w.r.t ‘x’ we get

d 2 y dy  dy 
1 ( y   ) 2     0
dx dx  dx 
2
d 2 y  dy 
1 ( y   ) 2     0
dx  dx 

d2y   dy  2 
 ( y   ) 2   1    
dx   dx  
  dy  2   d 2 y 
 ( y   )   1     1/ 2 
  dx    dx 

dy   dy  2   d 2 y   dy 
From (2) we get, ( x   )  ( y   )   1     1/ 2   
dx   dx    dx   dx 

Now by substituting the values of ( x   ) and ( y   ) in the given equation (1), we get
25

2 2
  dy  2   dy  2   dy  2 
1       1    
  dx    dx    dx  
2
 2
 r2
d y
2
d y
2

 2  2
 dx   dx 
2 2
  dy  2   dy  2   dy  2 
2
2d y
2

1        1      r  2 
  dx    dx    dx    dx 
2
  dy  2    dy  2 
2
2d y
2

1     1      r  2 
  dx     dx    dx 
3
  dy  2 
2
2d y
2

1      r  2 
  dx    dx 

which is the required solution

2. Form the differential equation by eliminating m from y = mx.

Solution: Given that y  mx  (1)

dy
Differentiating (1) w.r.t ‘x’ we get  m  (2)
dx

y
From (1) we get m  , therefore (2) becomes
x
dy y dy
 (or ) x  y, which is the required equation.
dx x dx

3. Eliminate a and b from xy  ae x  be x

Solution: Given that xy  ae x  be x (1)

Differentiating (1) w.r.t ‘x’ , we get

xy  ae x  be x
dy
x  y (1)  ae x  be x
dx

Differentiating again w.r.t ‘x’, we get

d 2 y dy dy
x 2  (1)   ae x  be x
dx dx dx

d2y dy
x 2
 2  ae x  be x (2)
dx dx

d2y dy
From (1) and (2), we get x 2
 2  xy, which is the required equation.
dx dx
26

Check Your Progress:

Form the differential equation by eliminating the arbitrary constants from

1. y  cx  c  c 3
2. y  c( x  c) 2
3. y 2  m(a 2  x 2 )

Answers:
3
 dy  dy
1.    ( x  1)  y  0
 dx  dx
2
 dy   dy 
2.    4 xy    8 y 2  0
 dx   dx 
2
d2y  dy   dy 
3. xy 2  x    y    0
dx  dx   dx 
27

UNIT- 3
EQUATIONS OF THE FIRST ORDER AND OF HIGHER DEGREE

Objective:

After successful completion of this unit students will be able to solve the equations in
different types( solvable for p, y, x and Clairaut’s form)

The equation of the first order and of nth degree is of the form

dy
p n  P1 p n 1  P2 p n  2  ....  Pn 1 p  Pn  0 ( where p  )
dx

This equation can be solved by any one of the following four methods.

1.3.1 TYPE A : Equation solvable for p

If the given equation is of nth degree, then express the given equation can be resolved into
factors and express in the form

( p  R1 )( p  R2 )( p  R3 )...( p  Rn )  0

We get n equations of the first order and of first degree. That is

( p  R1 )( p  R2 )( p  R3 )...( p  Rn )  0
p  R1  0, p  R2  0, ... p  Rn  0

Let the solutions of these n components be

1 ( x, y, c1 )  0, 2 ( x, y, c2 )  0,..., n ( x, y, cn )  0

Without loss of generality, we can replace all the constants by c.

Therefore the general solution for the given equation is 1 ( x, y, c1 ).2 ( x, y, c2 )....n ( x, y, cn )  0

1. Solve x 2 p 2  3xyp  2 y 2  0

Solution: Given that x 2 p 2  3xyp  2 y 2  0  (1)

By solving for p, we get

3 xy  9 x 2 y 2  4( x 2 )( y 2 )
p
2 x2
3xy  xy

2 x2
y 2 y
 or
x x
y 2 y
(i.e)., p  or p 
x x
28

dy
since p  we get
dx
dy y

dx x
dx dy
 
x y
 log x   log y  log c  log x  log y  log c  log xy  log c  xy  c

dy 2y
Next , 
dx x
dy 2dx
 
y x
 log y  2 log x  log c
 log y   log x 2  log c
 log y  log x 2  log c
 log yx 2  log c  yx 2  c

The solution is ( xy  c)( yx 2  c)  0

2. Solve p 2  5 p  6  0

Solution: Given that p 2  5 p  6  0


By solving for p, we get
5  25  4(1)(6)
p
2(1)
 3 or 2

dy
since p  we get
dx
dy
 3  dy  3dx  y  3x  c  y  3x  c
dx
dy
 2  dy  2dx  y  2 x  c  y  2 x  c
dx
The solution is ( y  3 x  c)( y  2 x  c)  0

1.3.2 TYPE B: Equation solvable for y:

The given equation f ( x, y, p)  0 can be put in the form y  g ( x, p )  (1)

 dp 
Differentiating w.r.t ‘x’ we get p    x, p,   (2)
 dx 
29

This equation is a first order differential equation in two variables p and x . Let the solution
of (2) will be of the form  ( x, p, c)  0  (3)

By eliminating p between (1) and (3), we get the required solution for the given equation.

3. Solve xp 2  2 yp  x  0

Solution: The given equation is xp 2  2 yp  x  0

xp 2  x x( p 2  1)
The given equation can be written as y    (1)
2p 2p

Differentiating w.r.t ‘x’ we get

dy 2 p  x(2 pdp / dx)  ( p  1)   x( p  1)2dp / dx


2 2


dx 4 p2
dp dp dp
p (4 p 2 )  4 xp 2  2 p 3  2 p  2 xp 2  2x
dx dx dx
4 p 3  2 p 3  2 p  2 x  p 2  1
dp
dx

2 p3  2 p  2 x
dx
 p  1
dp 2

2 p( p 2  1)  2 x  p 2  1
dp
dx

dp
px
dx
dx dp dp dx
 (or ) 
x p p x

On integrating, we get

log p  log x  log c


log p  log xc
p  cx  (2)

By eliminating p between equation (1) and (2) we get the required solution.

x(c 2 x 2  1) c 2 x 2  1
y 
2cx 2c
 2cy  c x  1
2 2
30

4. Solve y  xp  x4 p 2

Solution: Given that y  xp  x4 p 2

The given equation can be written as y   xp  x 4 p 2 (1)

Differentiating w.r.t ‘x’ we get

dy dp dp
 ( x  p )  ( x 4 2 p  p 2 4 x 3 )
dx dx dx
dp dp
p  x  p  x 4 2 p  p 2 4 x3  0
dx dx
 dp  3  dp 
 2 p  x   2x p  2 p  x   0
 dx   dx 
 dp 
 2 p  x  1  2 x p   0
3

 dx 
dp
either 2 p  x  0 or 1  2 x 3 p  0
dx

dp
If 2 p  x  0 , then
dx

dp dp 2dx dp 2dx
x  2 p     0
dx p x p x

log p  2 log x  log c


log p  log x 2  log c
log px 2  log c
c
px 2  c or p   (2)
x2

By substituting the result of p in (1) we get


2
c 4 c c
y  x 2
 x 4
 y    c 2  xy  c  c 2 x or xy  c 2 x  c  0
x x x

1.3.3 TYPE C: Equation solvable for x:

The given equation f ( x, y, p)  0 can be put in the form x  g ( y, p ) (1)

1  dp 
Differentiating w.r.t ‘y’, we get    y , p,   (2)
p  dy 

This equation is a first order differential equation in two variables p and y .


Let the solution of (2) will be of the form  ( y, p, c)  0 (3)
31

By eliminating p between (1) and (3), we get the required solution for the given equation.

5. Solve x  y 2  log p

Solution: Given that x  y 2  log p (1)

Differentiating w.r.t ‘y’, we get

dx 1 dp
 2y 
dy p dy
1 1 dp
 2y 
p p dy
dp dp
1  2 py  (or )  2 py  1
dy dy

This is linear in p, therefore the solution is


pe   (1) e dy  c  pe y   e y dy  c  (2)
2 ydy 2 ydy 2 2

By eliminating p between equation (1) and (2), we get the required solution.

6. Solve p 2  2 xp  1  0

Solution: Given that p 2  2 xp  1  0 (1)

By solving for x, equation (1) can be written as

p 2  1  2 xp
p2  1 p 1
x  
2p 2 2p

Differentiating w.r.t ‘y’ we get

dx 1 dp 1 dp
  2
dy 2 dy 2 p dy
1 dp  1  1 dp  p 2  1  1  p2 1  p 1 
  1  2 
   2   dy    dp  dy     dp
p 2dy  p  p 2dy  p  2 p   2 2p 

p2 1
On integrating we get, we get y   log p  c (2)
4 2

By eliminating p between (1) and (2), we get the required solution.


32

1.3.4 TYPE D: Clairaut’s Form:

This equation is of the form y  px  f ( P) .

This equation is solvable for y. So differentiating w.r.t ‘x’, we get

dy dp dp
 p(1)  x  f ' ( P)
dx dx dx
dp
 x  f ( P)   0
dx
dp
 either  0 or x  f ( P)  0
dx
dp
Here  0, gives p  c
dx

Therefore the solution for the given equation is y  cx  f (c) .

Note: If the given equation is of Clairaut’s form, we get the general solution for the given
equation by substituting p = c.

7. Solve y  ( x  a) p  p 2

Solution: Given that y  ( x  a) p  p 2  px  pa  p3

That it the given equation is of the form y  px  f ( p), which is of Clairaut ' s form

Differentiating w.r.t ‘x’ we get

dy dp dp dp
 p  x  a  3 p2
dx dx dx dx
dp
p  p  ( x  a  3 p2 )
dx
dp
( x  a  3 p2 )  0
dx
dp
(i.e)  0 or x  a  3 p 2  0
dx

dp
If  0 , then p  c . Therefore the given becomes.
dx

So, if the given equation is of Clairaut’s form, directly by substituting p = c we get the
general solution.

8. Solve xp 2  yp  p 2  1  0

Solution: Given that xp 2  yp  p 2  1  0 .


33

The above equation can be written as

yp  xp 2  p 2  1
1
y  xp  p 
p
 1
(i.e) y  xp   p   , which is of Clairaut ' s form
 p

Therefore, by substituting p = c in the given equation, we get the general solution.

That is xc 2  yc  c 2  1  0

Check Your Progress

Solve the following equations:

1. p 2  3 p  2  0
2. x(1  p 2 )  1
3. y 2  a 2 (1  p 2 )
4. p 3  4 xyp  8 y 2  0
5. y  xp 2  p
6. xp 2  yp  p 2  1  0

Answers:

1. ( y  x  c)( y  2 x  x)  0
2. y  sin 1 x  x(1  x)  c
 y2  a2 y 
3. x   a log   c
 a a 

4. 64 y  c(4 x  c) 2
5. ( p  1) 2 x  c  p  log p
6. cy  cp 2  c 2  1
34

BLOCK-II
UNIT-4
LINEAR EQUATIONS WITH CONSTANT COEFFICIENTS

Objectives:
After successful completion of this unit, the students will be able to

 Understand the concept of linear differential equation with constant coefficient

 Find the Particular Integral for the given equation of various functions on the right hand
side of the equation

 Find the solutions of this differential equation.

2.1 Linear Equations With Constant Coefficients

Introduction:
A linear equation is one in which the dependent variable y and its derivatives of any order
occur only in the first degree and are not multiplied together, their coefficients being
constants or functions of the independent variable x.

The general form of this equation is

dny d n 1 y d n2 y
 P1 n 1  P2 n 2  ...  Pn y  R( x)
dx n dx dx

Where P1 , P2 ,...Pn and R( x) are functions of x or constants.

Here the General Solution is the sum of Complementary Function and the Particular Integral.

(i.e.,) y  C.F  P.I

If the above equation is of second degree, then the equation will be of the form

d d2
(i.e.) D2 y  PDy  Qy  R( x) since D  , D2  2
dx dx

Method of finding Complementary Function:

Form the Auxiliary Equation (A.E) as am2  bm  c  0

b  b2  4ac
The above equation can be solved by using m
2a
35

Let the roots be m1 and m2.

Nature of the roots Complementary Function (yc)

Roots are real and distinct


1 yc = Ae m1x  Be m2 x
m1 and m2
Roots are real and equal
2 yc = ( Ax  B)emx
m1 = m2 ( = m)
Roots are imaginary
3 (i.e.)   i  yc = e x  A cos  x  B sin  x

If three Roots are real and distinct


4 yc = Aem1x  Bem2 x  Ce m3 x
m1,m2 and m3
If three Roots are real and equal
5 yc = ( Ax 2  Bx  C )emx
m1= m2 = m3 (= m)
If the two Roots are imaginary and
6 the third root is real yc = e x  A cos  x  B sin  x  Cem3x
(i.e.) m1,m2 =   i  and m3

1. Solve ( D2  5D  4) y  0

Solution: Let the A.E is m2  5m  4  0


Here a  1 b  5 c  4

b  b 2  4ac (5)  52  4(1)(4) 5  9 5  3


m     4,1 or 1, 4
2a 2(1) 2 2
i.e., the roots are real and distinct.
 y  Ae x  Be4 x

2. Solve ( D 2  4 D  4) y  0

Solution: Let the A.E is m 2  4m  4  0

Here a  1 b  4 c  4

b  b 2  4ac 4  42  4(1)(4) 4  0 4  0
m      2, 2
2a 2(1) 2 2
i.e., the roots are real and equal.
 y  ( Ax  B)e 2 x
36

3. Solve ( D 2  4 D  13) y  0

Solution: Let the A.E is m2  4m  13  0

Here a  1 b  4 c  13

b  b 2  4ac 4  42  4(1)(13) 4  36 4  6i


m      2  3i
2a 2(1) 2 2
i.e., the roots are real and imaginary. (i.e., )    2 and   3
 y  e 2 x ( A cos 3x  B sin 3x)

4. Solve ( D2  a 2 ) y  0

Solution: Let the A.E is m2  a 2  0

(i.e., ) m2  a 2  0  m 2   a 2  m    a 2  0  ai
i.e., the roots are real and imaginary. (i.e., )   0 ,   a
 y  e0 x ( A cos ax  B sin ax)
(i.e., ) y  A cos ax  B sin ax

5. Solve ( D3  3D2  6 D  8) y  0

Solution: Let the A.E is m3  3m2  6m  8  0

First we can find one of the root by using trial and error method.

  m   m3  3m 2  6m  8
 1  13  3(1) 2  6(1)  8  0,  m  1 is one of the root

Now we can reduce the third degree equation to second degree by using synthetic division

1 3 6 8
0 1 2 8 Now the equation reduces to m2  2m  8  0
1 2 8 0

Now the equation reduces to m2  2m  8  0


37

Here a  1 b  2 c  8

b  b 2  4ac (2)  (2) 2  4(1)(8) 2  36


m    4, 2
2a 2(1) 2
i.e., the three roots are 1, 4,  2
 y  Ae x  Be 4 x  Ce 2 x

6. Solve ( D2  9) y  0

Solution: Let the A.E is m 2  9  0

(i.e., ) m2  9  0  m 2  9  m   9  3, 3
i.e., the roots are real and distinct.
 y  Ae3 x  Be3 x

Check your Progress:

1. ( D3  3D2  4) y  0

2. ( D2  3D  2) y  0

3. ( D3  12D  16) y  0

4. ( D4  4D3  8D2  8D  4) y  0

5. ( D2  5D  6) y  0

Answers:

1. y  ( Ax  B)e2 x  Ce x

2. y  Ae x  Be2 x

3. y  ( Ax  B)e2 x  Ce4 x

4. y  e x ( Ax  B)(C cos x  D sin x)

5. y  Ae2 x  Be3 x
38

UNIT – 5

VARIOUS TYPES OF PARTICULAR INTEGRAL

General method of finding of finding Particular Integral (P.I):

Type I : If R(x) = 𝑒 𝑎𝑥

1
Particular Integral yp = eax
f ( D)

1 ax
= e (if f ( a ) ≠ 0 )
f (a)

Note: For eax , put D = a


1
Case(i) : If f ( a ) = 0 , then yp = x. eax
f ( D)

(i.e. multiply the Nr. with x and differentiate the Dr. w.r.t D )

1 ax
= x. e (if f ( a ) ≠ 0)
f (a)

1
Case(ii) : If f ( a ) = 0 , then yp = x 2 . eax
f ( D)

1
= x2 . eax (if f ( a ) ≠ 0)

f (a)

7. Solve ( D 2  2D  1) y  e3 x

Solution: Let the A.E is m 2  2m  1  0

Here a  1 b  2 c  1

b  b 2  4ac (2)  (2) 2  4(1)(1) 2  0


m   1, 1
2a 2(1) 2
i.e., the roots are real and equal
ComplimentaryFunction (C.F ) is yc  ( Ax  B)e x

1
Particular Integral (P.I) = e3 x
D  2D  1
2
39

1
Put D  3,  P.I = e3 x
(3)  2(3)  1
2

1
 e3 x
4
1
General Solution is y  ( Ax  B)e x  e3 x
4

8. Solve ( D2  5D  6) y  e x

Solution: Let the A.E is m2  5m  6  0

Here a  1 b  5 c  6

b  b 2  4ac (5)  (5) 2  4(1)(6) 5  1


m     2,  3
2a 2(1) 2
i.e., the roots are real and distinct
ComplimentaryFunction (C.F ) is yc  Ae 2 x  Be 3 x

1
Particular Integral (P.I) = ex
D  5D  6
2

1
Put D  1,  P.I = ex
(1)  5(1)  6
2

1
 ex
12
1 x
General Solution is y  Ae2 x  Be3 x  e
12

d2y dy
9. Solve 2
 3  2 y  e2 x  5
dx dx

Solution: The given equation is ( D2  3D  2) y  e2 x  5

Let the A.E is m2  3m  2  0

Here a  1 b  3 c  2

b  b 2  4ac (3)  (3) 2  4(1)(2) 3  1 3  1


m      1,  2
2a 2(1) 2 2
i.e., the roots are real and distinct
ComplimentaryFunction (C.F ) is yc  Ae  x  Be 2 x
40

1
Particular Integral (P.I) = (e 2 x  5)
D  3D  2
2

1 1
 P.I = e 2 x  2 5e0 x
D  3D  2
2
D  3D  2
 P.I1  P.I 2
1
P.I1  e 2 x
D  3D  2
2

Put D =  2
1
 P.I1 = e 2 x
(2)  3(2)  2
2

1
 e2 x (condition fails )
0

 multiply the numerator with x and differentiate the Dr. w.r.t D


1 1 x 2 x
 P.I1 = x e 2 x = x e 2 x  e   xe 2 x
2D  3 2(2)  3 1
1 1 5
and P.I 2 = 5e0 x = 5e0 x 
D  3D  2
2
002 2
5
General Solution is y  Ae x  Be2 x   xe2 x 
2

d2y dy
10. Solve 2
 2  y  cosh 2 x
dx dx

Solution: The given equation is ( D2  2D  1) y  cosh 2 x

Let the A.E is m2  2m  1  0

Here a  1 b  2 c  1

b  b 2  4ac 2  (2) 2  4(1)(1) 2  0 2  0


m      1,  1
2a 2(1) 2 2
i.e., the roots are real and equal
ComplimentaryFunction (C.F ) is yc  ( Ax  B)e  x
41

1
Particular Integral (P.I) = cosh 2 x
D  2D  1
2

1  e2 x  e2 x 
 P.I =  
D2  2D  1  2 
1 1 1 
  e2 x  2 e 2 x 
2  D  2D  1
2
D  2D  1 
Put D=2 in the first term and D=  2 in the second term

1 1 1 
  e2 x  e2 x 
2  2  2(2)  1
2
(2)  2(2)  1
2

1  1 2 x 1 2 x  1  1 2 x 2 x 
 e  e   e e 
2  9 1  2 9 
1 1 
General Solution is y  ( Ax  B)e  x   e 2 x  e 2 x 
2 9 

Check your Progress:

1. ( D2  2mD  m2 ) y  emx

2. (3D2  D  14) y  13e2 x

3. ( D2  13D  12) y  e2 x  5e x

4. ( D3  3D2  4D  2) y  e x

5. ( D2  4D  5) y  e2 x

Answers:

1. y  ( Ax  B)e2 x  Ce x

7
 x
2. y  Ae2 x  Be 3
 xe2 x

1 2 x 5 x
3. y  Ae x  Be12 x  e  xe
42 11

4. y  e x ( A cos x  B sin x)  Ce x  xe x

1
5. y  Ae  x  Be5 x  e 2 x
9
42

Type II : If R(x) = sin ax or cos ax

1
Particular Integral yp = sin ax (or cos ax )
f ( D)

Note: For Sinax or Cosax, put D 2   a 2

Case (i): If Denominator contains first degree in D (i.e., aD + b ), then multiply and divide

by its conjugate. (i.e., aD  b). Again put D 2   a 2

Case (ii): If Denominator is 0, then multiply the Nr. with x and differentiate the Dr. w.r.t D.

1
(i.e.) yp = x. sin ax (or cos ax )
f ( D)

( then the result reduces to either case (i) or case (iii)

1
Case (iii) : If Denominator contains only D i.e., sin ax ( or cos ax ),
D

1
then we can use sin ax ( or cos ax ) =  sin ax (or cos ax)dx
D

11. Solve ( D2  3D  2) y  sin 3x

Solution: Let the A.E is m2  3m  2  0

Here a  1 b  3 c  2

b  b 2  4ac (3)  (3) 2  4(1)(2) 3  1 3  1


m     2,1 or 1, 2
2a 2(1) 2 2
i.e., the roots are real and distinct
ComplimentaryFunction (C.F ) is yc  Ae x  Be 2 x

1
Particular Integral (P.I) = sin 3 x
D  3D  2
2

Here put D2   (a 2 ) i.e., D2   (32 )   9


43

1
 P.I = sin 3 x
9  3D  2
1 1
 sin 3x  sin 3x
3D  7 (3D  7)
1 (3D  7)
 sin 3x (i.e., Multiply and Divideby the conjugate of Dr.)
(3D  7) (3D  7)
(3D  7) (3D  7) (3D  7) (3D  7)
 sin 3x   sin 3x   sin 3x  sin 3x
(9 D  49)
2
(9(9)  49) 130 130

1 1
 3D(sin 3x)  7 sin 3x   3(cos 3x)(3)  7 sin 3x 
130 130
1
 9 cos 3x  7 sin 3x 
130

1
General Solution is y  Ae x  Be 2 x  9 cos 3x  7 sin 3x 
130

12. Solve ( D2  4) y  cos 2 x

Solution: Let the A.E is m 2  4  0

 m 2  4  m   4  m  2i
 m  0  2i
i.e.,   0 and   2

 Complimentary Function (C.F) is y c  e0 x ( A cos 2 x  B sin 2 x)  A cos 2 x  B sin 2 x

1
Particular Integral (P.I) = cos 2 x
D 4
2

Here put D2   (a 2 ) i.e., D 2   (22 )   4

1
 P.I = cos 2 x
4  4
1
 cos 2 x (condition fails)
0
 Multiply the numerator with x and differentiate the denominator w.r.t D
x x 1  x x  sin 2 x  1
 cos 2 x   cos 2 x    cos 2 xdx     x sin 2 x
2D 2 D  2 2 2  4

1
The General Solution is y  A cos 2 x  B sin 2 x  x sin 2 x
4
44

13. Solve ( D4  2D2 n2  n4 ) y  cos mx

Solution: Let the A.E is m 4  2m 2 n 2  n 4  0

 (m 2  n 2 )2  0
 m 2  n 2  0 twice
 m 2  n 2 twice
 m    n 2  m   in twice
i.e.,   0 and   n

 C. F is y c  e0 x ( A cos nx  B sin nx)(C  Dx)  ( A cos nx  B sin nx)(C  Dx )

1
Particular Integral (P.I) = cos mx
( D  n2 )2
2

Here put D2   (m2 )

1
 P.I = cos mx
(m  n 2 ) 2
2

1
= cos mx
(n  m2 )2
2

1
 y  ( A cos nx  B sin nx)(C  Dx)  cos mx
(n  m2 ) 2
2

14. Solve ( D2  4D  3) y  sin 3x cos 2 x

Solution: Let the Auxiliary Equation is m 2  4m  3  0

(4)  (4) 2  4(1)(3) 4  4 4  2


m    3,1( or 1 and 3)
2(1) 2 2
C.F is yc  Ae x  Be3 x

1
Particular Integral (P.I) = sin 3x cos 2 x
D  4D  3
2

1
since sin A cos B  sin( A  B)  sin( A  B)
2

1
sin 3x cos 2 x  sin 5 x  sin x 
2

1 sin 3 x cos 2 x
 P.I =
D  4D  3
2
2
45

1 1  1 1 
  2  sin 5 x   2  sin x
2  D  4D  3  2  D  4D  3 

1 1  1 1 
   sin 5 x    sin x
2  25  4 D  3  2  1  4 D  3 

1 1  1 1 
   sin 5 x    sin x
2  4 D  22  2  4 D  2 

1 1  1 1 
   sin 5 x    sin x
4  2 D  11  4  2D 1 
1 2 D  11 1 2D  1
 sin 5 x  sin x
4 (2 D  11)(2 D  11) 4 (2 D  1)(2 D  1)
1 2 D  11 1 2D  1
 sin 5 x  sin x
4 (4 D  121)
2
4 (4 D 2  1)
1 2 D  11 1 2D  1
 sin 5 x  sin x
4 (100  121) 4 (4  1)

2 D  11 2D  1
 sin 5 x  sin x
884 20
2 D(sin 5 x)  11(sin 5 x) 2 D(sin x)  sin x
 
884 20
2(5cox5 x)  11(sin 5 x) 2(cos x)  sin x
 
884 20
10cox5 x  11sin 5 x 2 cos x  sin x
 
884 20

10cox5 x  11sin 5 x 2 cos x  sin x


The General Solution is y  Ae x  Be3 x  
884 20

15. Solve ( D2  16) y  e3 x  cos 4 x

Solution: Let the Auxiliary Equation is m 2  16  0

 m 2  16  m   16  m  4i


 m  0  4i
i.e.,   0 and   4

 Complimentary Function (C.F) is y c  e0 x ( A cos 4 x  B sin 4 x)  A cos 4 x  B sin 4 x

1
Particular Integral (P.I) = (e 3 x  cos 4 x)
D  16
2
46

1 1
 e 3 x  2 cos 4 x
D  16
2
D  16
1 1
= e 3 x  2 cos 4 x
D  16
2
D  16
 P.I1  P.I 2

1 1 1
Let P.I1  e3 x  e3 x  e3 x
D  16
2
(3)  16
2
25

1 1 1
Let P.I2  cos 4 x  cos 4 x  cos 4 x (condition fails)
D  162
(4)  16
2
0

1 x 1  x x  sin 4 x  x sin 4 x
Therefore, P.I 2  x cos 4 x   cos 4 x    cos 4 xdx   
2D 2 D  2 2 4  8

1 3 x x sin 4 x
Hence P.I = e 
25 8

1 3 x x sin 4 x
 y  A cos 4 x  B sin 4 x  e 
25 8

Check your Progress:

1. ( D2  3D  2) y  sin 3x

2. ( D2  16) y  e3 x  cos 4 x

3. ( D2  2D  8) y  4cos 2 x

Answers:

1
1. y  Ae x  Be 2 x   9 cos 3x  7 sin 3x 
130

1 3 x x
2. y  A cos 4 x  B sin 4 x  e  sin 4 x
25 8

1
3. y  Ae 4 x  Be  x  (3cos 2 x  sin 2 x)
10
47

Type III : If R( x)  eaxV , where V is any function of x.

1
Particular Integral yp = eaxV
f ( D)
1
= eax . V
f ( D  a)

(Now the problem reduces to type II or type III )

16. Solve ( D2  4 D  13) y  e2 x cos3x

Solution: Let the A.E is m2  4m  13  0

Here a  1 b  4 c  13

b  b 2  4ac (4)  (4) 2  4(1)(13) 4  36 4  6i


m     2  3i
2a 2(1) 2 2
i.e., the roots are real and imaginary. (i.e., )   2 ,   3
ComplimentaryFunction (C.F ) is yc  e 2 x ( A cos 3x  B sin 3x)

1
Particular Integral (P.I) = e 2 x cos 3x
D  4 D  13
2

1
 P.I =e2 x cos 3x i.e., D  ( D  2)
( D  2)  4( D  2)  13
2

1
 e2 x cos 3x
D  4 D  4  4 D  8  13
2

1
 e2 x cos 3x ( Now the problem reduces to Type : II )
D 9
2

 we can put D 2  (a 2 ), i.e., D 2  (32 )   9


1 1
 P.I  e 2 x cos 3x  e 2 x cos 3x (condition fails )
9  9 0
 Multiply the numerator with x and differentiate the denominator w.r.t D
x 1  x  sin 3x  1 2 x
   e x sin 3x 
x
 P.I  e 2 x cos 3x  e 2 x  cos 3x   e 2 x 
2D 2 D  2 3  6

General Solution is y  e2 x ( A cos 3x  B sin 3x) 


6
 e x sin 3x 
1 2x
48

17. Solve ( D2  2D  2) y  e x sin x

Solution: Let the A.E is m 2  2m  2  0

Here a  1, b  2, c  2

b  b 2  4ac (2)  (2)2  4(1)(2) 2  4 2  2i


m    1  i
2a 2(1) 2 2
i.e., the roots are real and imaginary. (i.e., )   1 ,  1
ComplimentaryFunction (C.F ) is yc  e x ( A cos x  B sin x )

1
Particular Integral (P.I) = e  x sin x
D  2D  2
2

1
 P.I =e x sin x i.e., D  ( D  1)
( D  1)  2( D  1)  2
2

1
 e x sin x
D  2D  1  2D  2  2
2

1
 e x sin x ( Now the problem reduces to Type : II )
D  4D  5
2

 we can put D 2  (a 2 ), i.e., D 2  (12 )   1


1 1 1
 P.I  e  x sin x  e  x sin x  e  x sin x
1  4 D  5 4 D  4 4( D  1)
e x ( D  1)
 sin x (i.e., Multiply and Divideby the conjugate of Dr.)
4 ( D  1)( D  1)
e  x ( D  1)
 sin x
4 ( D 2  1)
e  x ( D  1)
 sin x i.e., by sub D 2  (a 2 ), i.e., D 2  (12 )   1
4 (1  1)
e x e x e x
 ( D  1) sin x   D (sin x )  sin x   cos x  sin x 
8 8 8
e x
General Solution is y  e x ( A cos x  B sin x)  cos x  sin x 
8
49

Check your Progress:

1. ( D2  4D  13) y  e2 x cos3x.

2. ( D3  2D  4) y  e x cos x.

3. ( D2  2 D  2) y  e x cos x.

Answers:

xe2 x sin 3x
1. y  e ( A cos 3x  B sin 3x) 
2x

xe x
2. y  Ae2 x  e x ( B cos x  C sin x)  (3sin x  cos x)
20

e x x sin x
3. y  e x ( A cos x  B sin x) 
2

Type IV : If R(x) = X (where Xis of the form x m , m being a positive integer)

1
Particular Integral yp = xm
f ( D)
1
= xm
1   ( D)

= 1   ( D )  x m
1

Now expand 1   ( D)  by using binomial expansion: (1  x)1  1  x  x 2  x3  ....


1

Note: (1  x)1  1  x  x 2  x3  ....

(1  x)2  1  2 x  3x 2  4 x3  ....

(1  x)2  1  2 x  3x 2  4 x3  ....

18. Solve ( D3  D2  D  1) y 1 x 2

Solution: Let the A.E is m3  m 2  m  1  0

  m   m3  m 2  m  1
 1  13  (1) 2  (1)  1  0,  m  1 is one of the root

Now we can reduce the third degree equation to second degree by using synthetic division
50

1 1 1 1
1
0 1 0 1 Now the equation reduces to m2  1  0
1 0 1 0

m2  1 0  m2  1 0  m2  1  m  1and  1

The three roots are 1, 1 and -1

 yc  ( Ax  B)e x  Ce x

1
Particular Integral (P.I) = (1  x 2 )
D  D2  D  1
3

1
 (1  x 2 )
1  ( D  D 2  D3 )
 1  ( D  D 2  D 3 )  1 (1  x 2 )
 1  ( D  D 2  D 3 )  ( D  D 2  D 3 ) 2  (1  x 2 )
(by omitting D 3 and higher powers D )
 1  D  D 2  D 2  (1  x 2 )
 1  D  2 D 2  (1  x 2 )
 (1  x 2 )  D(1  x 2 )  2 D 2 (1  x 2 )
 1  x 2  2 x  2(2)
 x2  2 x  5

 y  ( Ax  B)e x  Ce x  x 2  2 x  5

Check your Progress:

1. ( D2  2D  1) y  x 2  1

2. ( D2  1) y  2  5 x

3. ( D 2  4) y  x 2

Answers:

1. y  e x ( Ax  B)  x 2  4 x  7

2. y  Ae x  Be x  2  5x.

1 1
3. y  A cos 2 x  B sin 2 x  ( x 2  )
2 2
51

Type V : If R(x) = x sin ax (or) x cos ax

1
(i) For x cos ax use Real Part of xeiax
f ( D)
1
(ii) For x sin ax use Img. Part of xeiax
f ( D)

19. Solve ( D2  1) y  x cos x

Solution: Let the A.E is m 2  1  0

m2  1  0  m2  1  m   i

Complimentary Function y c  e0 x ( A cos x  B sin x)  A cos x  B sin x

1
Particular Integral (P.I) = x cos x
D 12

1
 Real Part of xeix
D 1
2

1
 R.P of eix x
( D  i)2  1
1
 R.P of eix 2 x
D  2 Di  i 2  1
1
 R.P of eix x
2 Di  D 2
1
 R.P of eix x
 D
2 Di 1  
 2i 
1
1  D
 R.P of e 1   x
ix

2 Di  2i 

1  D D2 
 R.P of e ix
1   x
2 Di  2i 4i 2 
1  1 1 D
 R.P of eix    x
2i  D 2i 4 

1  x2 x 1 
 R.P of eix    
2i  2 2i 4 
i  x2 x 1 
 R.P of  eix    
2  2 2i 4 
52

 x 2i x i 
 R.P of  eix    
 4 4 8
 x 2i x i 
 R.P of  (cos x  i sin x)    
 4 4 8
 x 2i cos x x cos x i cos x x 2 sin x ix sin x sin x 
 R.P of       
 4 4 8 4 4 8 
x cos x x 2 sin x sin x
  
4 4 8

x cos x x 2 sin x sin x


 y  A cos x  B sin x   
4 4 8

20. Solve ( D2  1) y  x sin x

Solution: Let the A.E is m 2  1  0

m2  1  0  m2  1  m  1,  1

Complimentary Function y c  Ae x  Be  x

1
Particular Integral (P.I) = x sin x
D 1
2

1
 Img. Part of xeix
D 1
2

1
 I .P of eix x
( D  i)2  1
1
 I .P of eix 2 x
D  2 Di  i 2  1
1
 I .P of eix 2 x
D  2 Di  2
1
 I .P of eix x
 D 2  2 Di 
2  1  
 2 

1
eix  D 2  2 Di 
 I .P of  1   x
2  2 
eix
 I .P of  1  Di  x
2
eix
 I .P of   x  iD( x) 
2
eix
 I .P of   x  i 
2
53

1
 I .P of  (cosx+isinx)  x  i 
2
1
 I .P of   x cos x  i cos x  ix sin x  sin x 
2
1
   cos x  x sin x 
2

1
 y  Ae x  Be  x   cos x  x sin x 
2
54

BLOCK-III

UNIT-6

LINEAR DIFFERENTIAL EQUATION WITH VARIABLE COEFFICIENTS

An equation is of the form


n 1 n2
dny n 1 d y n2 d y
x n
n
 p1 x n 1
 p2 x n2
 ...  pn y  X  (1)
dx dx dx
where p1 , p2 ,..., pn are constants and X is a function of x is called a homogeneous linear
equation of the nth order.

The equation (1) can be reduced to linear differential equation with constant coefficients by
substituting x  e z (or) z  log x .
dy dy dz 1 dy
 
dx dz dx x dz
dy dy d
x   Dy where D 
dx dz dz
d 2 y d  dy  d  1 dy  1  dy  1 d  dy 
Next ,        2    
dx 2
dx  dx  dx  x dz  x  dz  x dx  dz 
1  dy  1 d  dy  dz
    
x 2  dz  x dz  dz  dx
1  dy  1 d 2 y 1
  2  
x  dz  x dz 2 x
1  dy  1 d 2 y
  2   2 2
x  dz  x dz
d2y 1  dy  1 d 2 y
(ie)., 2   2    2 2
dx x  dz  x dz
d 2 y d 2 y  dy 
 x2  2     D2 y  Dy   D2  D  y  D  D  1 y
dx 2
dz  dz 
dy d2y
Therefore, by substituting x  Dy and x 2 2  D( D  1) y and so on in (1) , then
dx dx
equation (1) becomes linear differential equations with constant coefficients.

d2y dy
1. Solve x 2
2
 2x  4 y  x4
dx dx
Solution: Given x D y  2 xDy  4 y  x 4
2 2

( x 2 D2  2 xD  4) y  x 4

First, we can convert the variable coefficient into constant coefficient by substituting
x  e z , therefore log x  z
55

d
 xD  D where D 
dz
x 2 D 2  D( D  1)
Therefore, the given equation becomes

( D( D  1)  2 D  4) y  e 4 z
( D2  D  2 D  4) y  e 4 z
( D2  3D  4) y  e4 z
 A.E is m 2  3m  4  0
3  9  4(1)(4) 3  5
(i.e., )   4,  1
2(1) 2
C.F  Ae 4 z  Be  z

1
P.I  e4 z
D  3 D  4
2

1
 2 e4 z
4  3(4)  4
1
z e4 z
2 D  3
1
z e4 z
2(4)  3
1
 z e4 z
5
Therefore, the complete solution is
1
y  Ae 4 z  Be  z  ze 4 z
5
since e  x and z  log x , e nz  x n , we get
z

1
y  Ax 4  Bx 1  x 4 log x
5

d3y 2
2 d y dy
2. Solve x3 3
 3 x 2
 x  y  x  log x
dx dx dx

d3y 2
2 d y dy
Solution: Given that x3 3
 3 x 2
 x  y  x  log x
dx dx dx
x D y  3x D y  xDy  y  x  log x
3 3 2 2

( x3 D3  3x 2 D2  xD  1) y  x  log x

First we can convert the variable coefficient into constant coefficient by substituting
x  e z , therefore log x  z
56

d
 xD  D where D 
dz
x 2 D 2  D( D  1)
x 3 D 3  D( D  1)( D  2)
Therefore, the given equation becomes

x3 D 3  3x 2 D 2  xD  1) y  x  log x
[ D( D  1)( D  2)  3D( D  1)  D  1) y  e z  z
( D3  1) y  e z  z
 A.E is m3  1  0

m  1 and m 2  m  m  0
1  1  4(1)(1) 1  i 3 1 3
m  1 and m    i
2(1) 2 2 2
z 3 
1
3
C.F  Ae  z  e 2  B cos z  C sin z
 2 2 
1
P.I  3 (e z  z )
D  1
1 1
 3 ez  3 z
D  1 D  1
1
 e z  (1  D3 ) 1 z
2
1 z
 e  (1  D3 ) z
2
1 z
 e  z  D3 ( z )
2
1
 ez  z
2

Therefore, the complete solution is


z  3  1 z
1
3
y  Ae  z  e 2  B cos z  C sin z   e  z
 2 2  2
since e z  x and z  log x , e nz  x n , we get
1
 3 3  x
y  Ax 1  x 2  B cos log x  C sin log x    log x
 2 2  2
d2y dy
3. Solve x 2 2  3x  5 y  sin(log x)
dx dx
d2y dy
Solution: Given that x 2 2  3x  5 y  sin(log x)
dx dx
x D y  3xDy  5 y  sin(log x)
2 2

x D
2 2
 3xD  5 y  sin(log x)
57

First we can convert variable coefficient into constant coefficient by substituting


x  e z , therefore log x  z
d
 xD  D where D 
dz
x 2 D 2  D( D  1)
Therefore, the given equation becomes

x D2 2
 3xD  5 y  sin(log x)
 D( D  1)  3D  5 y  sin z
( D2  4 D  5) y  sin z
 A.E is m 2  4m  5  0
4  16  4(1)(5) 4  6
(i.e., )   5,  1
2(1) 2
C.F  Ae  z  Be5 z
1
P.I  2 sin z
D  4 D  5
1
 sin z
(1)  4 D  5
2

1
 sin z
4 D  6
1
 sin z
2(2 D  3)
(2 D  3)
 sin z
2(2 D  3)(2 D  3)
(2 D  3)
 sin z
2  2 D2  9 
(2 D  3)
 sin z
2  4(1) 2  9 
1
  2 D(sin z )  3sin z 
26
1
  2 cos z  3sin z 
26

Therefore, the complete solution is


1
y  Ax 1  Bx5   2 cos(log x)  3sin(log x) 
26

d 2 y 1 dy 12log x
4. Solve  
dx 2 x dx x2

d 2 y 1 dy 12log x
Solution: Given that  
dx 2 x dx x2
58

d2y dy
x2 2
x  12 log x
dx dx
x 2 D 2 y  xDy  12 log x
x D 2 2
 xD  y  12 log x
First we can convert variable coefficient into constant coefficient by substituting
x  e z , therefore log x  z
d
 xD  D where D 
dz
x 2 D 2  D( D  1)
Therefore, the given equation becomes

 D( D 1)  D y  12 z


D2 y  12 z
A.E is m2  0  m  0, 0

Therefore C.F is y  ( Az  B)e0 z  Az  B


1
P.I  2 12 z
D
1  1 
 12  z 
D  D 
1  z2 
 12  
D  2 

6
D
z 
1 2

 z3 
 6    2z3
3

The Complete Solution is y  A log x  B  2(log x)3

NOTE: For the following problems, we can use the result

1 d
X  e  z  e z X dz ( where D  , x  e z , dx  e z dz )
D   dz

d2y dy
5. Solve x 2 2
 4x  2 y  ex
dx dx
d2y dy
Solution: Given that x 2 2  4 x  2 y  e x
dx dx
x D y  4 xDy  2 y  e x
2 2

( x 2 D 2  4 xD  2) y  e x
First we can convert the variable coefficient into constant coefficient by substituting
x  e z , therefore log x  z
59

d
 xD  D where D 
dz
x 2 D 2  D( D  1)
Therefore, the given equation becomes
[ D( D  1)  4 D  2] y  e x
[ D2  3D  2] y  e x
A.E is m2  3m  2  0  m  1 and m  2

C.F is Ae z  Be2 z  Ax 1  B x 2


1
P.I  2 ex
D  3 D  2
1
 ex
 
( D  1)( D  2)
By resolving into partial fraction, we get
1 A B A( D  2)  B( D  1)
  
( D  1)( D  2) ( D  1) ( D  2) ( D  1)( D  2)
so, 1  A( D  2)  B( D  1)
When D  1 we get A  1
When D  2 we get B  1
1 1 1
  
( D  1)( D  2) ( D  1) ( D  2)
1  1 1  x
P.I  ex    e
( D  1)( D  2)  ( D  1) ( D  2) 
1 1
P.I  ex  ex

( D  1) 
( D  2)
1 1
 ex  ex
D  1 D  2
1
 e  z  e z e x dz  e 2 z  e 2 z e x dz ( X  e  z  e z X dz )
D  
 e  z  e x e z dz  e 2 z  e x e 2 z dz
 e  z  e x dx  e 2 z  e x e z e z dz
 e  z e x  e 2 z  e x xdx
 e  z e x  e 2 z ( xe x  e x )
 x 1e x  x 2 ( xe x  e x )
 x 2 e x

The Complete Solution is y  Ax 1  B x 2  x 2e x


60

d2y dy 1
6. Solve x 2  3x  y 
dx 2
dx (1  x)2

1
Solution: Given x 2 D 2 y  3xDy  y 
(1  x)2

 x2 D2  3xD  1 y  (1 1x)2
First we can convert the variable coefficient into constant coefficient by substituting

x  e z , therefore log x  z
d
 xD  D where D 
dz
x D  D( D  1)
2 2

Therefore, the given equation becomes

1
[ D( D  1)  3D  1] y 
(1  x)2
1
[ D2  2 D  1] y 
(1  x)2
A.E is m2  2m  1  0
 (m  1)2  0
 m  1, 1
C.F is ( Az  B)e z  ( A log x  B) x 1

1 1
P.I 
D  2 D  1 (1  x) 2
2

1 1

( D  1) (1  x) 2
2

1  1 1 
  2 
( D  1)  ( D  1) (1  x) 
1  z z 1 
  e e dz 
( D  1)  (1  x) 2

1 1 1 
  
( D  1)  x (1  x) 2
dx 

1 1 1 
  
( D  1)  x (1  x) 
1 1
 x 1  dx
x (1  x)
61

1 1
 x 1   dx (by resolving into partial fraction)
x (1  x)
 x 1  log x  log(1  x) 
1  x 
 log  
x  1 x 
1  x 
The Complete Solution is y  ( A log x  B) x 1  log  
x  1 x 
Check your progress:

Solve the following differential equations:

d2y dy
1. x 2 2
 x  2 y  x2
dx dx
2
d y dy
2. x 2 2  3 x  4 y  cos(log x)
dx dx
2
d y dy
3. x 2 2  4 x  2 y  x log x
dx dx
2
d y dy log x sin(log x)  1
4. x 2 2  x  y 
dx dx x
2
d y dy
5. x 2 2  3 x  5 y  x 2 sin(log x)
dx dx

Answers:

x2
1. A cos( 2 log x)  B sin( 2 log x)) 
2
1
2. ( A log x  B ) x 2  3cos(log x)  4sin(log x)
25
1
3. Ax 2  Bx 1  x(6 log x  5)
36
1 22 4
4. x( A  B log x)   cos(log x)  sin(log x)
100 x 125 x 125 x
1
 log x{4 cos(log x)  3sin(log x)}
25 x
x2
5. x 2  A cos(log x)  B sin(log x)   (log x) cos(log x)
2
62

UNIT- 7

EQUATIONS REDUCIBLE TO THE LINEAR EQUATIONS

An equation of the form


n 1
dny n 1 d y
(ax  b) n
n
 p1 (ax  b) n 1
 ...  pn y  X
dx dx

where p1 , p2 ,..., pn are constants and X is a function of x. This equation can be reduced to
linear equations with constant coefficients by putting ax  b  e z . So that z  log(ax  b) .
d
If D  , then (ax  b) D  aD and (ax  b) 2 D 2  a 2 D( D  1)
dz

d2y dy
1. Solve (2 x  5)  (2 x  5)  8 y  6 x
2
2
dx dx

d2y dy
Solution: Given (2 x  5)  (2 x  5)  8 y  6 x
2
2
dx dx

1
By substituting 2 x  5  e z , then x  (e z  5)
2

Also log(2 x  5)  z
(2 x  5) D  2 D
(2 x  5) 2 D 2  4 D( D  1)

Therefore the given equation becomes,

1
4 D( D  1) y  6(2 D) y  8 y  6 (e z  5)
2
(4 D  16 D  8) y  3e  15
2 z

A.E is 4 m 2  16m  8  0
 m 2  4m  2  0
4  16  4(1)(2) 4  8
m   2 2
2 2
C.F  Ae(2 2)z
 Be(2 2)z
 A(2 x  5) 2 2
 B (2 x  5) 2 2

P.I 
1
4 D  16 D  8
2  3e z  15 
1 1
 3e z  15e0 z
 
4 D  16 D  8
2
 
4 D  16 D  8
2

1 1
 3e z  15e0 z
4(1)  16(1)  8 8
63

3 15
  ez 
4 8
3 15
  (2 x  5)  since e z  2 x  5
4 8
3 x 15 15 3 x 45
    
2 4 8 2 8

3x 45
Therefore, the Complete Solution is y  A(2 x  5) 2 2  B(2 x  5) 2 2
 
2 8

d2y 2 2 dy
2. Solve (1  x )    (1  x 2 ) y  3x by means of the substitution
2 3
2
2 x (1 x )
dx dx
x  tan  .

Solution: Using the substitution x  tan 

dy dy d dy dx d 1
  cos 2  ( x  tan  ,  sec 2     cos 2  )
dx d dx d d dx sec 
2

dy dy
  cos 2 
dx d
1 dy dy
 
cos 2  dx d
dy dy
 sec 2  
dx d
 1  tan 2   
dy dy
dx d

 1  x 2 
dy dy

d dx
d y d  dy  d 
2
2 dy  d  2 dy  dx
    1  x     1  x  
d 2
d  d  d  dx  dx  dx  d
d2y d  dy  dx
(i.e., ) 2   1  x 2  
d dx  dx  d

 d2y dy 
 1  x 2  2  (2 x)  1  x 2 
 dx dx 

d2y 2
(i.e)
d 2
 1  x 
2 2 d y

dx 2
 2 x 1  x 2 
dy
dx
2 2
 1  x 2  2  1  x 2   2 x 1  x 2 
d y 3 d y 2 dy

d dx 2
dx

d2y
Therefore, the given equation becomes 1  x  2  (1  x 2 ) y  3x
2

d
64

d2y 
 1  x 2   2  y   3x
 d 

d2y 3x 3 tan  3 tan  sin 


 y   3 cos 2   3sin  cos 
d 2
1  x 1  tan  sec 
2 2 2
cos 
2
d y
 2  y  3sin  cos 
d

By solving this we get,

A.E = m2  1 0  m2  1 m  i

So C.F = e0  A cos  B sin    A cos   B sin 

1 1  sin 2  3 1
P.I  3sin cos   2 3   sin 2
D 1
2
D 1  2  2 D 1
2

3 1 1 1
P.I    sin 2   sin 2   2 sin cos    sin cos 
 2  4  1 2 2

Therefore the complete solution is

y  A cos   B sin   sin cos 


x 1
x  tan  ,sin   ; cos  
1  x2 1  x2

Hence the solution for the given equation becomes

A Bx x 1
y  
1  x2 1  x2 1  x2 1  x2
1 x
y ( A  Bx) 
1  x2 (1  x 2 )
(1  x 2 ) y  ( A  Bx) 1  x 2  x

3. By means of the substitution cos x  t solve the differential equation


d2y dy
sin x 2  cos x  y sin 3 x
dx dx
dt
Solution: Using the given substitution cos x  t ,   sin x
dx
dy dy dt dy
Hence    sin x
dx dt dx dt
d y d  dy  d  dy
2
  dy d  dy  
      sin x     cos x  sin x   
dx 2
dx  dx  dx  dt   dt dx  dt  
65

d2y dy d dy  dt 
   cos x  sin x  
dx 2
dt dt dt  dx 
dy d2y
  cos x  sin x 2   sin x 
dt dt
dy d2y
  cos x  sin 2 x 2
dt dt
dy d2y
By substituting the result of and , in the given equation, we get
dx dx 2

 dy d2y   dy 
sin x   cos x  sin 2 x 2   cos x   sin x   y sin 3 x
 dt dt   dt 
dy d 2 y dy
 sin x cos x  sin 3 x 2  sin x cos x  y sin 3 x
dt dt dt
2
d y
sin 3 x 2  y sin 3 x
dt
2
d y y sin 3 x
 2 
dt sin 3 x
d2y d2y
 2  y  2 y0
dt dt
A.E = m2  1 0  m2  1 m  1

So C.F = y  Aet  Bet

Therefore the complete solution is y  Aecos x  Be cos x

Check Your Progress:


Solve the following Differential equations:
d2y dy
1. ( x  a)
2
2
 4( x  a)  6 y  x
dx dx
2
d y dy
2. x 2 2  2 x(2 x  3)  3( x 2  4 x  2) y  0 by means of the substitution yx 3  t
dx dx
2
d y dy
3. (3 x  2) 2 2  3(3 x  2)  36 y  3 x 2  4 x  1
dx dx
Answers:
3x  a
1. y  A( x  a) 2  B( x  a)3 
6
2. yx3  Ae3 x  B  x
1 1
3. A(3x  2) 2  B(3x  2) 2  (3x  2) 2 log(3x  2) 
108 108
66

UNIT-8

VARIATION OF PARAMETERS

This method is used to find the particular solution of a non-homogeneous equation when two
linearly independent solutions of the homogeneous equations are known.
Consider the linear equation
d2y dy
2
 p ( x)  q ( x) y  r ( x)
dx dx
If y1 ( x) and y2 ( x) are linearly independent solutions of the equation
d2y dy
2
 p( x)  q( x) y  0 , then
dx dx
The general solution is y  c1 y1 ( x)  c2 y2 ( x)
Using this method, we used to find the solution of the given equation by replacing the
constant c1 and c2 by the functions of x.

Here Complementary Function is

c1 f1  c2 f 2 (where c1 , c2 are constants and f1 , f 2 are functions of x )

Then Particular Integral is Pf1  Qf 2

f2 X f1 X
Where P   f  
dx and Q  f  
dx
1 f 2  f1 f 2 1 f 2  f1 f 2

d2y
1.Solve  y  tan x using method of variation of parameter.
dx 2

Solution: Let the A.E is m  1  0


2

 m 2  1
 m  0i
The C.F = e0 x (c1 cos x  c2 sin x)  c1 cos x  c2 sin x

f1  cos x f 2  sin x
f1   sin x f 2  cos x
 f1 f 2  f1f 2   cos x  cos x     sin x  sin x   cos 2 x  sin 2 x 1

Here P.I= Pf1  Qf 2

f2 X f1 X
where P    dx and Q   dx
f1 f 2  f1 f 2 f1 f 2  f1 f 2
sin x tan x sin x 1  cos 2 x  1 
 P   dx    sin x dx    dx      cos x dx
(1) cos x cos x  cos x 
 P    sec xdx   cos xdx   log(sec x  tan x)  sin x
67

cos x tan x cos x.sin x


Q dx   dx   sin xdx  cos x
(1) cos x
Therefore P.I= sin x  log(sec x  tan x)  (cos x)  ( cos x)(sin x)
 cos x sin x  cos x log(sec x  tan x)  cos x sin x
  cos x log(sec x  tan x)
Hence the complete solution is y  c1 cos x  c2 sin x  cos x log(sec x  tan x)

d2y
2. Solve 2
 n2 y  sec nx using method of variation of parameter.
dx
Solution: Let the A.E is m  n  0
2 2

 m2  n 2
 m  0  in
The C.F = e0 x (c1 cos nx  c2 sin nx)  c1 cos nx  c2 sin nx

f1  cos nx f 2  sin nx
f1  n sin nx f 2  n cos nx
 f1 f 2  f1f 2   cos nx  n cos nx    n sin nx  sin nx   n cos 2 nx  n sin 2 nx  n

Here P.I= Pf1  Qf 2

f2 X f1 X
where P    dx and Q   dx
f1 f 2  f1 f 2 f1 f 2  f1 f 2
sin nx sec nx 1 1 1 1
 P   dx    sin nx dx    tan nxdx   2 logsec nx
n n cos nx n n
cos nx sec nx 1 x
Q dx   dx 
n n n
 1  x
Therefore P.I=  2 logsec nx  (cos nx)  (sin nx)
 n  n
1  x
  2 logsec1 nx  (cos nx)  (sin nx)
n  n
cos nx x
 2
log cos nx  sin nx
n n
cos nx x
Hence the complete solution is y  c1 cos nx  c2 sin nx  2
log cos nx  sin nx
n n
d2y dy
3. Solve ( x  1) 2  x  y  ( x  1)2 given that x and ex are the particular integrals of
dx dx
the equation without the right hand member.

Solution: Given that x and ex are the particular integrals of the equation without the right
hand member.
Therefore the general solution is y  Pf1  Qf 2  Px  Qe
x

f2 X f1 X
where P    dx and Q   dx
f1 f 2  f1 f 2 f1 f 2  f1 f 2
68

f1  x f2  ex
f1  1 f 2  e x
 f1 f 2  f1f 2   x   e x   1   e x   xe x  e x  e x ( x  1)
e x ( x  1)
 P   dx    dx   x
e x ( x  1)
x( x  1)
Q x dx   e x xdx   xe x dx  ( x)(e x )  (1)(e x )   e x ( x  1)
e ( x  1)
Therefore the general solution is
y = Pf1 +Qf 2 =   x  x+  e -x (x +1) e x =  x 2  x  1

Hence the complete integral is y  c1 x  c2 e  ( x  x  1)


x 2

Check your progress:

Solve the following equations using method of variation of parameters.


d2y
1. 2  y  cosec x
dx
d2y
2. 2  4 y  tan 2 2 x
dx
d2y
3. 2  y  x cos x
dx
d 2 y dy
4. 2   e x sin x
dx dx
2
d y
5. 2  4 y  4sec 2 2 x
dx

Answers:
1. y  c1 cos x  c2 sin x  x cos x  (log sin x ) sin x
1 1
2. y  c1 cos 2 x  c2 sin 2 x  sin 2 x log(sec 2 x  tan 2 x) 
4 2
2
x 1 x
3. y  c1 cos 2 x  c2 sin 2 x  cos x  sin x  sin x
4 8 4
x
e
4. y  c1  c2 x  (sin x  cos x)
2
5. y  c1 cos 2 x  c2 sin 2 x  sin 2 x log(sec 2 x  tan 2 x)  1
69

UNIT-9

METHOD OF UNDETERMINED COEFFICIENT

To find the P.I. of f(D)y = X , we assume a trial solution containing unknown constants
which are determined by substitution in the given equation. The trial solution to be assumed
in each case, depends on the form of X. Thus

Term in X Choice of Yp

Ae px Ce px

Ax n an x n  an 1 x n 1  ...  a1 x  a 0
A cos px a1 cos px  a1 sin px

A sin px a1 cos px  a1 sin px

1. Solve  D 2  D  2  y 10 cos x using method of undetermined coefficients

1  1  4(1)(2) 1  3
Solution: Let A.E is m 2  m  2  0  m    2, 1
2 2

C.F = c1e x  c2e2 x

Let us assume the trial solution of P.I as

y  a1 cos x  a2 sin x
Dy   a1 sin x  a2 cos x
D 2 y   a1 cos x  a2 sin x

By substituting these values in the given equation, we get

(a1 cos x  a2 sin x)   a1 sin x  a2 cos x   2(a1 cos x  a2 sin x)  10 cos x
 a1 cos x  a2 sin x  a1 sin x  a2 cos x  2a1 cos x  2a2 sin x  10 cos x
 a1 cos x  a2 sin x  a1 sin x  a2 cos x  2a1 cos x  2a2 sin x  10 cos x
 3a1  a2  cos x   a1  3a2  sin x  10 cos x
By equating the coefficient of cosx and sinx, we get

3a1  a2 10 and a1  3a2  0

By solving these equation, we get a1  3 and a2   1

Hence the complete solution is y  c1e x  c2e 2 x  3cos x  sin x


70

2. Solve  D 2  4 D  3 y 10 cos x using method of undetermined coefficients.

4  16  4(1)(3) 4  2
Solution: Let A.E is m2  4m  3  0  m    3,1
2 2

C.F = c1e x  c2 e3 x

Let us assume the trial solution of P.I as

y  Ce 2 x
y  2Ce 2 x
y  4Ce 2 x

By substituting these values in the given equation, we get

 4Ce   4  2Ce   3  Ce   10e


2 x 2 x 2 x 2 x

 15Ce2 x  10e 2 x

By equating the coefficient of e 2 x , we get 15C=10, which gives C=2/3

2
Hence the complete solution is y  c1e x  c2e3 x  e 2 x
3

Check Your Progress:

Solve the following equations using method of undetermined coefficients:

d2y dy
1. 2  2  4 y  2 x 2  3e  x
dx dx
2
d y
2. 2  4 y  8 x 2
dx
d2y dy
3. 2  2  y  e x  x
dx dx

Answers:

1 1
1. y  e x (c1 cos 3x  c2 sin 3x)  x 2  x  e  x
2 2
2. y  c1 cos 2 x  c2 sin 2 x  2 x  1
2

1 2 x
3. y   c1  c2 x  e x  x e x2
2
71

BLOCK – IV

UNIT -10
dx dy dz
SIMULTANEOUS DIFFERENTIAL EQUATION OF THE FORM  
P Q R

Let us consider the equations involving more than two variables, only one of which is
the independent variable. A set of such equation is called a set of ordinary simultaneous
equations. By taking z as the independent variable and x and y as the pair of dependent
variables, a pair of simultaneous differential equations of the first order and first degree may
be written as

dx dy
P1  Q1  R1  0
dz dz
dx dy
P2  Q2  R2  0
dz dz
where P1 , P2 , Q1 , Q2 , R1 , R2 arethe functions of x, y and z.

These equations can be written as

Pdx
1  Q1dy  R1dz  0
P2 dx  Q2 dy  R2 dz  0
the ratios of the differentials dx, dy and dz can be obtained as
dx dy dz
  (or )
Q1 R2  Q2 R1 R1 P2  R2 P1 PQ 1 2  P2Q1

dx dy dz
  where P, Q and R are functions of x, y and z
P Q R

The genera solution for this equation is u  c1 and v  c2 or we can represent this solution as
 (u, v)  0 where  is an arbitrary function.

Method 1. Method of Grouping

dx dy dz
1. Solve the equations  
yz xz xy

Solution: By taking the first two ratios, we get

dx dy dx dy
 (i.e.,)   xdx  ydy  xdx  ydy  0
yz xz y x

On integrating, we get
72

x2 y 2
  c  x 2  y 2  2c  c1
2 2
(i.e., ) x 2  y 2  c1

By taking the first and the last ratios, we get

dx dz dx dz
 (i.e.,)   xdx  zdz  xdx  zdz  0
yz xy z x

On integrating, we get

x2 z 2
  c  x 2  z 2  2c  c2
2 2
(i.e., ) x 2  z 2  c2

Therefore the general solution is  ( x 2  y 2 , x 2  z 2 )  0

dx dy dz
2. Solve the equations  2  2
x  yz y  xz z  xy
2

Solution: From the first two ratios, we get

dx  dy dy  dz dz  dx
 2 2  2
( x  y )  yz  xz ( y  z )  xz  xy ( z  x 2 )  xy  yz
2 2

dx  dy dy  dz dz  dx
  
( x  y )( x  y )  z ( x  y ) ( y  z )( y  z )  x( y  z ) ( z  x)( z  x)  y ( z  x)
d ( x  y) d ( y  z) d ( z  x)
  
( x  y )( x  y  z ) ( y  z )( x  y  z ) ( z  x)( x  y  z )
d ( x  y ) d ( y  z ) d ( z  x)
  
( x  y) ( y  z) ( z  x)

By taking the first two ratios, and integrating, we get

d ( x  y) d ( y  z)
 ( x  y)

( y  z)
 log( x  y )  log( y  z )  log c
 log( x  y )  log( y  z )  log c
x y
 log  log c
yz
x y
  c1
yz

yz
Similarly, by taking the last t two ratios, and integrating, we get  c2
zx
73

 x y yz
Therefore, the general solution is   , 0
 yz zx

dx dy dz
3. Solve the equations 2
 2  2 2 2
y x x y z

Solution: From the first two ratios, we get

dx dy
2
 2  x 2 dx  y 2 dy  x 2 dx  y 2 dy  0
y x

x3 y 3 x3  y 3
On integrating, we get  c   c  x3  y 3  3c  c1
3 3 3

(i.e.,) x3  y 3  c1

dx dz dz
Next, by taking first and the last ratio, we get 2
 2 2 2 which gives x 2 dx  2  0
y x y z z

x3 1
on integrating we get,   c2
3 z

 x3 1 
Therefore, the general solution is   x3  y 3 ,    0
 3 z

dx dy dz
4. Solve the equations  
yz zx x y

Solution: The given equation can be written as

dx  dy dy  dz dz  dx dx  dy  dz
  
yx zy xz 2( x  y  z )

By using the first two ratios, we get

dx  dy dy  dz d ( x  y) d ( y  z)
  
yx zy ( x  y) ( y  z)

On integrating we get,

log( x  y )  log( y  z )  log c1


log( x  y )  log( y  z )  log c1
 x y x y
log    log c1   c1
 yz yz

By using the first and the last , we get


74

dx  dy dx  dy  dz d ( x  y ) dx  dy  dz
  
yx 2( x  y  z ) ( x  y) 2( x  y  z )

Thus, we have

dx  dy  dz d ( x  y)
2 0
( x  y  z) ( x  y)
 log( x  y  z )  2 log( x  y )  log c2
 log( x  y  z )  log( x  y ) 2  log c2
 ( x  y  z )( x  y ) 2  c2

 x y 
Therefore, the general solution is   , ( x  y  z )( x  y ) 2   0
 yz 

Method 2. Method of Multipliers

We have to find the multipliers, l , m, n and u, v, w such that one of the equations

dx dy dz ldx  mdy  ndz udx  vdy  wdz


   
P Q R lP  mQ  nR uP  vQ  wR

can be easily integrated.

dx dy dz
5. Solve  
x( y  z ) y ( z  x) z ( x  y )

Solution: By using 1,1,1 as multipliers,

dx  dy  dz dx  dy  dz
each ratio = 
x( y  z )  y ( z  x)  z ( x  y ) 0

Thus we have dx  dy  dz  0

On integrating , we get x  y  z  c1

1 1 1
Similarly by using the multipliers , ,
x y z

1 1 1 1 1 1
dx dy dz dx  dy  dz
y x y z
each ratio = x   z 
( y  z ) ( z  x) ( x  y ) 0

1 1 1
Thus we have dx  dy  dz  0
x y z

On integrating, we get log x  log y  log z  log c2  log xyz  log c2  xyz  c2

Thus the general solution is  ( x  y  z, xyz )  0


75

dx dy dz
6. Solve  
mz  ny mx  lz ly  mx

Solution:

By choosing the multiplier x, y and z we get

xdx  ydy  zdz xdx  ydy  zdz


Here each ratio  
x(mz  ny )  y (nx  lz )  z (ly  mx) 0

 xdx  ydy  xdz  0.


x2 y 2 z 2 x2  y 2  z 2
On integrating we get    c c
2 2 2 2
 x 2  y 2  z 2  2c  c1

Next, by choosing the multiplier l , m and n in (1), we get

ldx  mdy  ndz ldx  mdy  ndz


Here each ratio  
l (mz  ny )  m(nx  lz )  n(ly  mx) 0

 ldx  mdy  ndz  0.


On integrating we get lx  my  nz  c 2

Hence the general solution is  ( x2  y 2  z 2 , lx  my  nz)  0

Check your Progress:

Solve the simultaneous equations:

dx dy dz
1.  2 
xy y x( yz  2 x)
dx dy dz
2.   2
z( x  y) z( x  y) x  y 2
dx dy dz
3.   2
y  xz yz  x x  y 2
dx dy dz
4.  
x z y
dx dy dz
5.  
x( y  z ) y ( z  x ) z ( x  y 2 )
2 2 2 2 2
76

Answers:

 x ye x 
1.   , 2 0
 y y  2x 
2.   x 2  y 2  z 2 , 2 xy  z 2   0
3.   xy  z, x 2  y 2  z 2   0
 yz
4.   y 2  z 2 , 0
 x 
5.   xyz, x 2  y 2  z 2   0
77

UNIT- 11

nth ORDER EXACT DIFFERENTIAL EQUATIONS

A differential equation of order n of the form

 d n y d n1 y dy 
f  n , n1 ,..., , y   X ( x)  (1)
 dx dx dx 
is said to be exact if it can be derived by differentiating directly , a differential equation of the
 d n1 y dy 
next lower order of the form f1  n1 ,..., , y    Xdx  c (2)
 dx dx 

Equation (2) is said to be the first integral of equation (1).

Condition for the exactness of the linear differential equation:

Let us consider the differential equation

dny d n 1 y d n2 y
P0  P  P  ...  Pn y  X  (3)
dx n 1 dx n 2
1 2
dx n

where P0 , P1 ,..., Pn are functions of x only.

Here Pn  (1) Pn(1)


1  ( 1) Pn  2  ...  ( 1) P0
2 (2) n (n)
 0 is the condition for the given equation
(3) to be exact. If this condition is satisfied then the equation
d n1 y d n2 y
P0 n 1   P1  P0  n 2   P2  P1 P0''  ...   Pn 1  Pn2  ...  (1) n 1 P0n 1  y   g ( x)dx

dx dx

is the first integral of the equation (1)

d2y dy
1. Solve x 2  2 x  2y  0
dx dx

Solution: Here P0  x, P1  2 x , P2  2

The condition of exactness for a second order equation is P2  P1 P0 0

P0  x, P1  2 x P2  2
P1  2 P2  0  P2  0

 P2  P1 P0  2  2  0  0

Which shows that the given equation is exact.

Next, the first integral of the given second order equation is obtained from
78

dy
P0  ( P1  P0) y  C1
dx

dy
Thus, x  (2 x  1) y  c1
dx
dy  1 1
  2   y  c1
dx  x x

dy
Which is linear equation of the first order form  Py  Q
dx

Hence the solution is y(e )   Qe


Pdx Pdx
dx  c2  (1)

1
 Pdx   (2  x )dx  2 x  log x
e2 x
e
Pdx 1
 e2 x log x  e2 x e log x  e2 x elog x  e2 x x 1 
x

e2 x 1 e
2x
 (1)  y    c1  dx  c2
x x  x
e2 x
y  c1  e2 x x 2 dx  c2
x

d3y 2

2. Show that the differential equation  x  4  3  9 x 2 d y dy


2
 18 x  6 y  6 is exact
3

dx dx dx
and find the first integral.

Solution: From the given we get P0  x3  4, P1  9 x 2 , P2 18 x, P3  6

The condition of exactness for a third order equation is P3  P2  P1 P0 0

P0  x 3  4 P1  9 x 2 P2  18 x P3  6
P0  3 x 2
P1  18 x P2 18
P0 6 x P1 18
P0'''  6

 P3  P2  P1 P0 (6)  (18)  (18)  (6)  0

which shows that the given equation is exact.

Next, the first integral of the given third order equation is obtained from

d2y dy
P0 2
 ( P1  P0)  ( P2  P1 P0'' ) y  c1
dx dx
79

d2y dy
( x3  4) 2
 (9 x 2  3x 2 )  (18 x  18 x  6 x) y  c1
dx dx
2
d y dy
( x3  4) 2  6 x 2  6 xy  c1
dx dx

d2y dy
(i.e.,) The first integral of the given equation is ( x3  4) 2
 6 x2  6 xy  c1
dx dx

d3y d2y dy
3. Solve (1  x  x ) 3  (3  6 x) 2  6 0
2

dx dx dx

Solution: From the given we get P0  1  x  x 2 P1  3  6 x P2  6 P3  0

The condition of exactness for a third order equation is P3  P2  P1 P0 0

P0  1  x  x 2 P1  3  6 x P2  6 P3  0
P0  2 x  1 P1  6 P2  0
P0 2 P1 0
P0'''  0

 P3  P2  P1 P0 (0)  (0)  (0)  (0)  0

which shows that the given equation is exact.

Next, the first integral of the given third order equation is obtained from

d2y dy
P0 2
 ( P1  P0)  ( P2  P1 P0'' ) y  c1
dx dx

d2y dy
(1  x  x 2 ) 2
 2(2 x  1)  2 y  c1
dx dx

Again by comparing the second order equation with the standard form we get

P0 1  x  x 2 P1  4 x  2 P2  2  ( x)  c1

The condition of exactness for a second order equation is P2  P1 P0 0

P0 1  x  x 2 P1  4 x  2 P2  2
P0  2 x  1 P1  4
P ''0  2

 P2  P1 P0 (2)  (4)  (2)  0

Which shows that the second order equation is also exact.

Hence the second integral of the given equation is obtained from


80

dy
P0  ( P1  P0) y    ( x)dx  c2
dx

dy
(1  x  x 2 )
 (2 x  1) y  c1 x  c2
dx
dy  2 x  1  c x  c2
  2 
y 1
dx  1  x  x  1  x  x 2
dy
Which is of the form  Py  Q
dx

c1 x 2  c2 x  c3
on simplification we get y 
x2  x  1

Check Your Progress:

Check the following differential equation is exact and solve.

d2y
  4 x 2  3 x   (2 x  3) y  0
dy
1. x3 2
dx dx
2
d y dy
2. 2 x 2 2  15 x  7 y  3 x 2
dx dx
3
d y d2y dy
3. x 3  ( x  3) 2  4 x  2 y  0
2

dx dx dx

Answers:
3

1. xy  c2 e x
 c1
x 2 c1
2. y   7  c2 x
9 x
dy
3. x  ( x 2  5) y  c1 x  c2
dx
81

BLOCK –V

UNIT- 12

PARTIAL DIFFERENTIAL EQUATIONS OF THE FIRST ORDER

An equation which contains partial derivatives, is called an partial differential equation.

The order of the partial differential equation is defined to be the order of the highest
partial derivative occurring in the equation.

The degree of the partial differential equation is the degree of the highest partial
derivatives occurring in the equation, after the equation has been made free from radicals and
fractions.

If z  f ( x, y ), then we denote the first order partial derivatives of z with respect to x and
y by p and q respectively.

z z
i.e., p  and q 
x y

The second order partial derivatives of z with respect to x and y are denoted as follows:

2 z 2 z 2 z
r , s  and t 
x 2 xy y 2

The general form of the first order partial differential equation is given by

f ( x, y, z, p, q)  0
82

CLASSIFICATION OF INTEGRALS

Complete Integral:

Let the partial differential equation be f ( x, y, z, p, q)  0  (1)

If we find the relation  ( x, y, z, a, b)  0  (2)


Where a and b are arbitrary constants, then the solution (2) is called the Complete Integral of
the given partial differential equation.

Particular Integral:

If  ( x, y, z, a, b)  0  (1) is the complete integral of the first order partial differential

equation f ( x, y, z, p, q)  0  (2). Then by giving the particular values to the constants a


and b in the complete integral, we obtain a solution to the given partial differential equation,
which is called a Particular Integral.

Singular Integral:
If  ( x, y, z, a, b)  0  (1) is the complete integral of the first order partial differential

equation f ( x, y, z, p, q)  0  (2). The equation of the envelope of the integral surfaces


represented by the complete integral (1) of the given equation (2) is called Singular Integral.
The singular Integral is obtained by eliminating the constants a and b from the equations
 
 ( x, y , z , a , b )  0  0 and 0
a b
General Integral:
We assume the arbitrary relation between a and b of the form b = f(a) in the complete

integral  ( x, y, z, a, b)  0 . Then this equation becomes   x, y, z, a, f (a)  0 .

By differentiating this equation partially with respect to a, we get


 
 f (a)  0
a b
The elimination of the constant a between these two equations is called the General Integral
of f ( x, y, z, p, q)  0 .
83

UNIT-13

DERIVATION OF PARTIAL DIFFERENTIAL EQUATION

The Partial Differential Equation (PDE) can be formed by

(i) Eliminating the arbitrary constants

(ii) Eliminating the arbitrary functions.

We can use the following notation for forming PDE

z z 2 z 2 z 2 z
p , q  ,r  2 , s  and t  2
x y x xy y

Eliminating the arbitrary constants

1. Eliminate a and b from z  axy  b

Solution: Given z  axy  b    (1)

z
Diff. (1) partially w.r.t x , we get  ay  p  ay    (2)
x

z
Diff. (1) partially w.r.t y , we get  ax  q  ax    (3)
y

(2) p ay
   px  qy (or ) px  qy  0
(3) q ax

2. Form the partial differential equation by eliminating a and b from

z  ( x 2  a)( y 2  b)

Solution: Given z  ( x 2  a)( y 2  b)    (1)

z
Diff. (1) partially w.r.t x , we get  2 x( y 2  b)  p  2 x( y 2  b)    (2)
x

z
Diff. (1) partially w.r.t y , we get  2 y ( x 2  a)  q  2 y ( x 2  a)    (3)
y

(2)  (3)  pq  4 xy( x 2  a)( y 2  b)


 pq  4 xyz  z  ( x 2  a)( y 2  b) 

3. Form the partial differential equation by eliminating a and b from

z  ( x  a)( y  b)

Solution: Given z  ( x  a)( y  b)    (1)


84

z
Diff. (1) partially w.r.t x , we get  ( y  b)  p  ( y  b)    (2)
x

z
Diff. (1) partially w.r.t y , we get  ( x  a)  q  ( x  a)    (3)
y

(2)  (3)  pq  ( x  a)( y  b)


 pq  z (or ) z  pq  z  ( x  a)( y  b)

4. Form the partial differential equation by eliminating a and b from

z  xy  y x 2  a 2  b2

Solution: Given z  xy  y x 2  a 2  b2 ------(1)

Diff. (1) partially w.r.t x , we get

z 1 
1
xy
 y  y ( x 2  a 2  b2 ) 2 2 x  p  y     (2)
x 2 x  a 2  b2
2

Diff. (1) partially w.r.t y , we get

z
 x  x 2  a 2  b 2  q  x  x 2  a 2  b 2    (3)
y
 q  x  x2  a 2  b2    (4)
xy
From (2) we get p  y     (5)
x  a 2  b2
2

(4)  (5)   q  x  p  y    
x2  a 2  b2  xy
 x a b
2 2 2

  xy

i.e.,  q  x  p  y   xy
 qp  qy  xp  xy  xy  0
 px  qy  pq  0

Eliminating the arbitrary functions

1. Form the partial differential equation by eliminating the arbitrary function from
z  f ( x2  y 2 )
Solution: Given z  f ( x2  y 2 ) ---- (1)

z
Diff. (1) partially w.r.t x , we get  f ( x 2  y 2 ) (2 x)
x

 p  f ( x 2  y 2 ) (2 x)  (2)
85

z
Diff. (1) partially w.r.t y , we get  f ( x 2  y 2 ) (2 y )
y

 q  f ( x 2  y 2 ) (2 y)  (3)

(2) p f ( x 2  y 2 )(2 x) p x
gives     py  xq  py  xq  0
(3) q f ( x  y )(2 y)
2 2
q y

2. Form the pde by eliminating f from z  f ( x, y)

Solution: Given z  f ( x, y ) ---- (1)

z
Diff. (1) partially w.r.t x , we get  f ( x, y )  p  f ( x, y )    (2)
x

z
Diff. (1) partially w.r.t y , we get  f ( x, y )  q  f ( x, y )    (3)
y

(2) p f ( x, y) p
gives    1  p  q (or ) p  q  0
(3) q f ( x, y ) q

3. Form the PDE by eliminating f from z  f ( x 2  y 2  z 2 )

Solution: Given z  f ( x 2  y 2  z 2 ) ---- (1)

z z
Diff. (1) partially w.r.t x , we get  f ( x 2  y 2  z 2 ) (2 x  2 z )
x x

 p  f ( x 2  y 2  z 2 ) (2 x  2 zp)  (2)

z z
Diff. (1) partially w.r.t y , we get  f ( x 2  y 2  z 2 ) (2 y  2 z )
y y

 q  f ( x 2  y 2  z 2 ) (2 y  2 zq)  (3)

(2) p f ( x 2  y 2  z 2 )(2 x  2 zp ) p 2 x  2 zp 2( x  zp )
gives    
(3) q f ( x  y  z )(2 y  2 zq )
2 2 2
q 2 y  2 zq 2( y  zq )
p x  zp
   p( y  zq )  q ( x  zp )
q y  zq
 py  pqz  qx  pqz  0
 py  qx  0

4. Eliminate f and  from z  f ( x  ay )   ( x  ay )

Solution: Given z  f ( x  ay )   ( x  ay ) (1)

Diff. (1) partially w.r.t x and y, we get


86

z
 f ( x  ay )   ( x  ay )
x

z
 a f ( x  ay )  a  ( x  ay )
y

Again differentiating these equation w.r.t x and y we get

2 z
 f ( x  ay )   ( x  ay )
x 2

2 z
 a 2 f ( x  ay)  a 2 ( x  ay)  a 2  f ( x  ay)   ( x  ay)
y 2

2 z 2  z
2
 2 a  t  a2r  t  a2r  0
y x 2

NOTE: If the given equation is of the form f (u, v)  0 or f (u )  v where u and v are
the functions of x,y and z. Then we find the pde by using
p q 1
ux uy uz  0
vx vy vz

5. Eliminate the arbitrary function f from f ( xy  z 2 , x  y  z )  0

Solution: The given equation is of the form f (u, v)  0


where u  xy  z 2 and v  x  y  z

u  xy  z 2  u x  y , u y  x, u z  2 z
v  x  y  z  vx  1, v y  1, vz  1

p q 1 p q 1
 ux uy u z  0  y x 2 z  0  p ( x  2 z )  q ( y  2 z )  1( y  x)  0
vx vy vz 1 1 1

 p( x  2 z )  q(2 z  y )  y  x
87

UNIT- 14

LAGRANGE’S METHOD OF SOLVING THE LINEAR EQUATION

A first order PDE of the form P( x, y, z ) p  Q( x, y, z )q  R( x, y, z ) is called Lagrange’s


Linear Equation.
dx dy dz
The Auxiliary Equation is given by   .
P Q R
Let the two independent integrals of these ordinary differential equations be
u  a and v  b. Then the general solution of this equation is given by  (u, v)  0 where u

and v are functions of x,y,z.


Note: We have to find two independent results u and v either by using Grouping Method or
Multiplier Method such that f (u, v)  0

1. Solve ( y  z ) p  ( z  x)q  x  y

Solution: Given that ( y  z ) p  ( z  x)q  x  y

The given equation is of the form Pp  Qq  R . Therefore it is of Lagrange’s type.

dx dy dz
 Its A.E is   -----(1)
yz zx x y

dx  dy  dz dx  dy  dz
Here each ratio  
( y  z )  ( z  x)  ( x  y ) 0

 dx  dy  dz  0.
On integrating we get x  y  z  c1

Next, by choosing the multiplier x, y and z in (1), we get

xdx  ydy  zdz xdx  ydy  zdz


Each ratio  
x( y  z )  y ( z  x)  z ( x  y ) 0

 xdx  ydy  zdz  0.


x2 y 2 z 2 x2  y 2  z 2
On integrating we get    c c
2 2 2 2
 x 2  y 2  z 2  2c  c2
Hence the general solution is f ( x  y  z , x 2  y 2  z 2 )  0
88

2. Solve x( y  z ) p  y ( z  x)q  z ( x  y )

Solution: Given that x( y  z ) p  y ( z  x)q  z ( x  y )

The given equation is of the form Pp  Qq  R . Therefore it is of Lagrange’s type.

dx dy dz
 Its A.E is   -----(1)
x( y  z ) y ( z  x) z ( x  y )

dx  dy  dz dx  dy  dz
Here each ratio  
x( y  z )  y ( z  x)  z ( x  y ) 0

 dx  dy  dz  0.
On integrating we get x  y  z  c1

1 1 1
Next, choosing the multiplier , and in (1), we get
x y z

1 1 1 1 1 1
dx  dy  dz dx  dy  dz
x y z x y z
Each ratio  
1 1 1 0
x( y  z )  y ( z  x)  z ( x  y )
x y z

1 1 1
dx  dy  dz  0.
x y z
On integrating we get log x  log y  log z  log c  log( xyz )  log c  xyz  c 1
Hence the general solution is f ( x  y  z , xyz )  0

3. Solve (mz  ny ) p  (nx  lz )q  ly  mx

Solution: Given that (mz  ny ) p  (nx  lz )q  ly  mx

The given equation is of the form Pp  Qq  R . Therefore it is of Lagrange’s type.

dx dy dz
 Its A.E is   -----(1)
mz  ny nx  lz ly  mx

By choosing the multiplier x, y and z in (1), we get

xdx  ydy  zdz xdx  ydy  zdz


Here each ratio  
x(mz  ny )  y (nx  lz )  z (ly  mx) 0

 xdx  ydy  xdz  0.


89

x2 y 2 z 2 x2  y 2  z 2
On integrating we get    c c
2 2 2 2
 x 2  y 2  z 2  2c  c1

Next, by choosing the multiplier l , m and n in (1), we get

ldx  mdy  ndz ldx  mdy  ndz


Here each ratio  
l (mz  ny )  m(nx  lz )  n(ly  mx) 0

 ldx  mdy  ndz  0.


On integrating, we get lx  my  nz  c 2

Hence the general solution is f ( x2  y 2  z 2 , lx  my  nz )  0

4. Solve ( y  z ) p  ( z  x)q  x  y

Solution: Given that ( y  z ) p  ( z  x)q  x  y

The given equation is of the form Pp  Qq  R . Therefore it is of Lagrange’s type.

dx dy dz
 Its A.E is   -----(1)
yz zx x y

dx  dy dy  dz dz  dx
Here each ratio =  
( y  z )  ( z  x) ( z  x)  ( x  y ) ( x  y )  ( y  z )

dx  dy dy  dz dz  dx
i.e., each ratio =   ----(2)
yx zy xz

By considering the first two ratio’s we get,

dx  dy dy  dz dx  dy dy  dz d ( x  y ) dy  dz
    
yx zy x y yz x y yz
On integrating we get log( x  y )  log( y  z )  log c1
log( x  y )  log( y  z )  log c1
 x y x y
 log    log c1   c1
 yz yz

dx  dy  dz
From (1), by adding all the ratio’s we get, each ratio  ----(3)
2( x  y  z )

Next, by taking the first ratio in (2) and the ratio in (3), we get
90

dx  dy dx  dy  dz 2d ( x  y ) d ( x  y  z ) d ( x  y  z) d ( x  y)
    2 0
yx 2( x  y  z ) ( x  y ) ( x  y  z) ( x  y  z) ( x  y)
On integrating we get log( x  y  z )  2 log( x  y)  log c
 log( x  y  z )  log( x  y ) 2  log c
 log ( x  y  z ) ( x  y ) 2   log c  ( x  y  z ) ( x  y ) 2  c2
x y 
Therefore the general solution is f  , ( x  y  z ) ( x  y)2   0
yz 
5. Solve ( x2  yz ) p  ( y 2  zx)q  ( z 2  xy)

Solution: The given equation is of the form Pp  Qq  R . Therefore it is of Lagrange’s type.

dx dy dz
 Its A.E is  2  2 -----(1)
x  yz y  zx z  xy
2

Each ratio is equal to

dx  dy dy  dz dz  dx
 2 2  2
x  y  yz  zx y  z  zx  zy z  x 2  xy  yz
2 2

d ( x  y) d ( y  z) d ( z  x)
 
( x  y  z )( x  y ) ( x  y  z )( y  z ) ( x  y  z )( z  x)

By equating the first two ratios and on integrating we get

d ( x  y) d ( y  z ) x y
  log( x  y)  log( y  z )  log c  c
( x  y) ( y  z) yz

dx  dy  dz
Also each ratio =  (2)
x  y  z 2  xy  yz  zx
2 2

By using the multiplier x,y,z and by adding all the ratios,

xdx  ydy  zdz


Also each ratio =  (3)
x  y 3  z 3  3xyz
3

From (2) and (3) we get

dx  dy  dz xdx  ydy  zdz



x  y  z  xy  yz  zx ( x  y  z )( x 2  y 2  z 2  xy  yz  zx)
2 2 2
91

xdx  ydy  zdz


 d ( x  y  z) 
x yz
 ( x  y  z )d ( x  y  z )  xdx  ydy  zdz
on integrating we get
( x  y  z )2 x 2 y 2 z 2
    c
2 2 2 2
x  y  z  2 xy  2 yz  2 zx x 2 y 2 z 2
2 2 2
    c
2 2 2 2
 xy  yz  zx  c

 x y 
Therefore the general solution is   , xy  yz  zx   0
 yz 
92

UNIT – 15

CHARPIT’S METHOD

Let us discuss Charpit’s method for solving the first order partial differential equation
f ( x, y, z, p, q)  0 . Let the Chartpit’s equation be
dx dy dz dp dq
   
f p f q pf p  qf q  ( f x  pf z )  ( f y  qf z )

Select any two proper fractions so that the resulting integral may be the simplest relation
involving at least one of p and q. Then the simplest relation of these step is solved along with
the equation to determine p and q. By substituting these values of p and q in dz  pdx  qdy
which is on integration gives the complete integral of the given equation.
1. Solve pxy  pq  qy  yz

Solution: Let the given equation can be written as f  pxy  pq  qy  yz  (1)


Let the Charpits Auxiliary Equations are
dx dy dz dp dq
     (2)
f p f q pf p  qf q  ( f x  pf z )  ( f y  qf z )

Since f  pxy  pq  qy  yz

f x  py f y  px  q  z fz   y f p  xy  q fq  p  y

dx dy dz dp dq
Therefore equation (2) becomes    
xy  q p  y xyp  pq 0 px  q  qy

By equating the last two ratios , we get dp  0, this implies p  a (constant)

By substituting p =a in equation (1) we get axy  aq  qy  yz

 aq  qy  yz  axy
 q (a  y )  y ( z  ax)
y ( z  ax)
q
ya

 dz  pdx  qdy
y ( z  ax)
dz  adx  dy
ya
y ( z  ax)
dz  adx  dy
ya

d ( z  ax) y
 dy
z  ax ya
93

d ( z  ax)  a 
 1   dy
z  ax  a y
on integrating, we get
log( z  ax)  y  a log(a  y )  b

This is the complete integral. There is no singular integral.

The general integral can be found as usual.

2. Solve 2 xz  px 2  2qxy  pq  0

Solution: Let the given equation can be written as f  2 xz  px2  2qxy  pq  (1)
Let the Charpits Auxiliary Equations are
dx dy dz dp dq
     (2)
f p f q pf p  qf q  ( f x  pf z )  ( f y  qf z )

Since f  2 xz  px2  2qxy  pq

f x  2 z  px  2qy f y   2qx f z  2x f p   x2  q f q   2xy  p

dx dy dz dp dq
Therefore equation (2) becomes    
 x  q 2 xy  p 2 x 2 z  2qy 0
2

By equating the last two ratios , we get dq  0, this implies q  a (constant)

By substituting q =a in equation (1) we get 2 xz  px 2  2axy  pa  0

2 x( z  ay )
p
x2  a
dz  pdx  qdy
2 x( z  ay )
dz  dx  ady
x2  a
2 x( z  ay )
dz  ady  dx
x2  a

d ( z  ay ) 2x
 2 dx
( z  ay ) x a
on integerating we get
log( z  ay )  log( x 2  a )  log b
 z  ay  b( x 2  a )

which is the complete integral


94

3. Solve  p 2  q 2  y  qz

Solution: Let the given equation can be written as f  p 2 y  q 2 y  qz  (1)


Let the Charpits Auxiliary Equations are
dx dy dz dp dq
     (2)
f p f q pf p  qf q  ( f x  pf z )  ( f y  qf z )

Since f  p 2 y  q 2 y  qz

fx  0 f y  p2  q2 fz   q f p  2 py f q  2qy  z

Therefore equation (2) becomes

dx dy dz dp dq
   
2 py 2qy  z 2 p  qz  2q y pq  p 2
2 2

By equating the last two ratios we get pdp  qdq  0  p 2  q 2  a 2

Using this in the given equation (1), we get

a2 y
a 2 y  qz  q 
z
since p 2 +q 2 =a 2 , p  a 2  q 2
a 2
(i.e) p  z  a 2 y2
z

dz  pdx  qdy we get z 2  a 2 y 2  (ax  b)2

which is the complete integral.


95

UNIT – 16

STANDARD FORMS

TYPE1: The equation is of the form f ( p, q )  0  (1)

Let the solution of this equation be z  ax  by  c  (2)

By differentiating (2) partially w.r.t x, we get p = a

By differentiating (2) partially w.r.t y, we get q = b

By substituting p=a and q=b in the equation (1) we get a relation connecting a and b. Here
we find b in terms of a and substitute in (2) which gives the complete solution

1. Solve pq  2

Solution: The given equation pq  2  (1) contains only p and q.


So which is of type 1. (i.e.,) f ( p, q )  0

Let us assume the solution of this equation be z  ax  by  c  (2)


Differentiating (2) partially w.r.t x we get p=a

Differentiating (2) partially w.r.t y we get q=b

By substituting p = a and q = b in the equation (1) we get ab = 2  b = 2/a

By substituting these values in (2), we get

2
z  ax  yc  (3) which is the complete integral
a

Differentiating partially with respect to c ,we get 0=1, which is absurd. Therefore there is no
singular solution.

To find the general integral, we can substitute c = f(a) in (3)

2
z  ax  y  f (a )
a

2
Differentiating partially with respect to a we get 0  x  y  f (a )
a2

By eliminating the constant ‘a’ between the last two equations, we get the General Integral.
96

2. Solve p 2  q 2  npq

Solution: The given equation p 2  q 2  npq  (1) contains only p and q.


So which is of type 1. (i.e.,) f ( p, q )  0

Let us assume the solution of this equation be z  ax  by  c  (2)


Differentiating (2) partially w.r.t x, we get p = a

Differentiating (2) partially w.r.t y, we get q = b

By substituting p=a and q=b in the equation (1) we get a 2  b 2  nab . Solving for b we get

b 2  nab  a 2  0
na  n 2 a 2  4(1)(a 2 )
b
2

b

a n  n2  4 
2

By substituting these values in (2), we get

z  ax 

a n  n2  4  yc  (3) which is the complete integral
2

Differentiating partially with respect to c ,we get 0=1, which is absurd. Therefore there is no
singular solution.

To find the general integral, we can substitute c = f(a) in (3)

z  ax 

a n  n2  4  y  f (a )
2

Differentiating partially with respect to a we get 0 x


n  n2  4  y  f (a)
2

By eliminating the constant ‘a’ between the last two equations, we get the General Integral.

TYPE2:

Here only one of the variables x,y,z occurs explicitly. Such equations can be written in
one of the forms f ( x, p, q)  0 ; f ( y, p, q)  0 ; f ( z, p, q)  0
Method of solving these type of problems:
97

Type Substitute Solve for Substitute the previous


f ( x, p , q )  0 qa p results in dz  pdx  qdy
f ( y , p, q )  0 pa q and on integrating, we get
the Complete Integral
f ( z , p, q )  0 q  ap p

3. Solve q  xp  p 2

Solution: The given equation is of the form f ( x, p, q)  0 .

So put q = a in the given equation and solve for p

a  xp  p 2 or p 2  xp  a  0
 x  x 2  4(1)( a )
p
2(1)
 x  x 2  4a
p
2

Therefore dz  pdx  qdy becomes


  x  x 2  4a 
dz    dx  ady
 2 
 
on integrating we get
x2  x 2  x 
z  x  4a  a sinh 1     ay  b
4 4  2 a 

4. Solve p  q  z

Solution: The given equation is of the form f ( z, p, q)  0 .

So put q = ap in the given equation and solve for p

z
 p  q  z becomes p  ap  z  p (1  a )  z  p 
1 a

Therefore dz  pdx  qdy becomes

dz  p  dx  ady 
z
dz   dx  ady 
1 a
dz 1
  dx  ady 
z 1 a

x  ay
On integrating we get log z  c
1 a
98

TYPE 3: Equations of the form f ( x, p)  g ( y, q)


For this type of problem, we can assume f ( x, p)  g ( y, q)  a and solve for p and q. Then
substitute in dz  pdx  qdy and on integrating we get the complete integral.

5. Solve p  q  x  y

Solution: The given equation can be written as p  x  y  q .

(i.e) The given equation is of the form f ( x, p)  g ( y, q)

we can assume f ( x, p)  g ( y, q)  a and solve for p and q.

px  yq  a
p  x  a and y  q  a
p  x  a and q  y  a

Therefore dz  pdx  qdy becomes dz  ( x  a)dx  ( y  a)dy

( x  a)2 ( y  a) 2
On integrating we get z    b , which is the complete integral
2 2

6. Solve p  q  sin x  sin y

Solution: The given equation can be written as p  sin x  sin y  q .

(i.e) The given equation is of the form f ( x, p)  g ( y, q)

we can assume f ( x, p)  g ( y, q)  a and solve for p and q.

p  sin x  sin y  q  a
p  a  sin x ; q  sin y  a

Therefore dz  pdx  qdy becomes dz  (a  sin x)dx  (sin y  a)dy

On integrating we get z  ax  cos x  cos y  ay  c , which is the complete integral

TYPE 4: Clairaut’s Form : The equation of the form z  px  qy  f ( p, q )

Here we get the complete integral directly by substituting p = a and q = b in the given
equation. For this type, singular integral exists.

7. Solve z  px  qy  2 pq

Solution: Given that z  px  qy  2 pq  (1)

which is of the form z  px  qy  f ( p, q )


That is the given equation is of Clairaut’s form. So by substituting p =a and q = b in the given
equation we get the complete integral.
99

The complete integral is z  ax  by  2 ab


Differentiating the complete integral partially with respect to a and b we get
1 b b
0  x  2 b. 0 x  x
2 a a a
1 a a
0  y  2 a. 0 y  y
2 b b b

By eliminating a and b from the above two equation we get the singular integral

 b  a
x y    
    xy  1
 a  b 

By substituting b=f(a) in the complete integral and differentiate partially with respect to a
and eliminate the constant a between the two equations, we get the general solution.
100

UNIT – 17

EQUATIONS REDUCIBLE TO THE STANDARD FORM

If the partial differential equation is not fall to any of the four standard types, then it can
be reduced to any one of the four types by change of variable.

(A) If ( x m p) and ( y n q) occur in the partial differential equation, then we can substitute

(i) x1m  X and y1n  Y if m  1 and n  1

(ii) log x  X and log y  Y if m  1 and n  1

(B) If ( z k p) and ( z k q) occur in the partial differential equation, then we can substitute ,

(i) z k 1  Z if k  1

(ii) log z  Z if k  1

1. Solve x 2 p 2  y 2 q 2  z 2

Solution: x p  y q  z (1)
2 2 2 2 2

This equation is not in the four standard form. So we can reduce the given equation in to
standard form by substituting log x  X and log y  Y

z z X z 1 1
p    P 
x X x X x x
 P  xp
Similarly Q  yq

By substituting these values in the equation, we get

P 2  Q2  z 2 ,  (2) whichis of type 3

dz dz dz dz
Let u  X  aY , so that P  and Q  a (  P,  Q)
du du dX dY

By substituting the results of P and Q in (2) we get

dz z
 
du 1  a2
dz du
 
z 1  a2

Integrating both sides, we get


101

1
log z  uc
1  a2
1
log z  ( X  aY )  c
1  a2
1
log z  (log x  a log y )  c
1  a2

2. Solve p 2  x 2 y 2 q 2  x 2 z 2

Solution: The given equation can be written as

p2  x2 y 2q 2  x2 z 2
2
 p
z      yq   (1)
2 2

x

Now we can substitute X  x and Y  log y


2

z z
p  2 x  P2x
x X
p
  2P
x
z z Y z 1 1
q   Q 
y Y x Y y  y
 Q  yq

Hence the equation (1) becomes z 2  4P 2  Q2

This equation is Type 2 . By solving this we get the solution as

(4  a 2 )(log z )2   x 2  a log y  b 
2

Check Your Progress:

Solve the following equations:

1. p 2  q 2  z 2 ( x 2  y 2 )
2. x 2 p 2  y 2 q 2  z
3. p 2 x 2  z ( z  qy )
4. x 2 p  y 2 q  z 2
102

Answers:

a2 1 x x 2 y a2 y
1. log z  sinh  a x  2
y  a  cosh 1  b
2 2

2 a 2 2 2 a
2. 4(1  a ) z  (log x  a log y  b)
2 2

3. 2 log z  (a  a 2  4)(log x  log y  b)


1 1 a
4. (1  a)    b
z x y

****
Document Information

Analyzed document Differential Equations(final version).pdf (D152427504)

Submitted 12/7/2022 6:29:00 AM

Submitted by

Submitter email kumarnano@gmail.com

Similarity 10%

Analysis address kumarnano.tnou@analysis.urkund.com

Sources included in the report

partial Differential Equation.pdf


45
Document partial Differential Equation.pdf (D142231462)

URL: https://archive.org/stream/in.ernet.dli.2015.449484/2015.449484.A-Textbook_djvu.txt
27
Fetched: 5/9/2022 5:38:42 PM

PDE.docx
4
Document PDE.docx (D120439796)

16699a0444.pdf
5
Document 16699a0444.pdf (D30528404)

URL: https://www.methodist.edu.in/web/uploads/files/ch4-1.pdf
18
Fetched: 11/22/2022 9:52:16 AM

URL: https://hoc360.net/wp-content/uploads/2018/06/ptrinh-va-bpt-logarit.pdf
5
Fetched: 1/14/2022 2:20:34 PM

15699A0554.docx
2
Document 15699A0554.docx (D21499327)

writeup for seminar 2.pdf


6
Document writeup for seminar 2.pdf (D139888065)

16691A0539delt.pdf
2
Document 16691A0539delt.pdf (D30457521)

Chewang Tenzin Doya (M.Ed Math).pptx


3
Document Chewang Tenzin Doya (M.Ed Math).pptx (D74940038)

PDE.pdf
7
Document PDE.pdf (D140277235)

15691A0284.docx
2
Document 15691A0284.docx (D21637273)

You might also like